You are on page 1of 58

Quiz 1: Strategic Management and Strategic Competitiveness

https://testbank45.com/wp-content/uploads/2018/06/9781133584698_TestBank_Chapter-1.pdf
1. Define strategic competitiveness and above-average returns. What is the relationship between
strategic competitiveness and returns on investment? (Xác định khả năng cạnh tranh chiến lược và lợi
nhuận trên trung bình. Mối quan hệ giữa năng lực cạnh tranh chiến lược và lợi tức đầu tư là gì?)
Strategic competitiveness is achieved when the firm successfully formulates and implements a value-
creating strategy. Above-average returns are returns in excess of what investors expect to earn from other
investments with similar risk levels. Firms will only be able to earn above-average returns if they develop a
competitive advantage. Competitive advantage derives from a strategy that competitors cannot duplicate or find
too costly to imitate.
2. Hypercompetition is a characteristic of the current competitive landscape. Define
hypercompetition and identify its primary drivers. How can organizations survive in a hypercompetitive
environment? (Hypercompetition là một đặc điểm của bối cảnh cạnh tranh hiện tại. Xác định siêu cạnh
tranh và xác định trình điều khiển chính của nó. Làm thế nào các tổ chức có thể tồn tại trong một môi
trường siêu cạnh tranh?)
Hypercompetition is a condition of rapidly escalating competition based on price-quality positioning,
competition to create new knowledge and establish first-mover advantage, and competition to protect or invade
established product or geographic markets. In hypercompetition, firms aggressively challenge their competitors.
Markets are assumed to be inherently unstable and changeable. The two primary drivers of hypercompetition
are the global economy and rapid technological change. To survive in a hypercompetitive environment firms
need strategic flexibility. This demands continuous
learning which allows the firm to develop new skills so that they can adapt to the changing environment
and to consistently engage in change.
3. Describe the industrial organization (I/O) model of above-average returns. What are its main
assumptions? What is the key to success according to the I/O model?
The I/O model of above-average returns argues that the external environment is the primary determinant of
firm success, rather than the firm's internal resources. The model has 4 underlying assumptions:
1) The external environment is assumed to impose pressures and constraints that determine the strategies
that would result in above-average returns.
2) Most firms competing within a particular industry, or in a certain segment of the industry, are assumed
to control similar strategically relevant resources and pursue similar strategies in light of those resources.
3) Resources used to implement strategies are mobile across firms, which results in resource differences
between firms being short-lived.
4) Organizational decision makers are assumed to be rational and committed to acting in the firm's best
interests as shown by their profit maximizing behaviors.
The key to success according to the I/O model is to find the most attractive industry (the one with the
highest profit potential) in which to compete.
4. Describe and discuss the resource-based model of above-average returns.
The resource-based model focuses on the firm's internal resources and capabilities. These resources and
capabilities determine the firm's strategy and its ability to earn above-average returns. The firm's resources are
inputs into its production process. Resources must be formed into capabilities, the capacity to perform a task or
activity in an integrative manner. According to this model, capabilities evolve over time and must be managed
dynamically to achieve above-average returns. Resources and capabilities that give a firm a competitive
advantage are called core competencies. This model assumes that resources are not highly mobile across firms;
consequently, all firms within a particular industry may not possess the same strategically relevant resources
and capabilities. So, different firms will have different core competencies. The organization's strategy is based
on finding the best environment in which to exploit its core competencies.
5. What are a firm's vision and mission? What is the value to the firm of having a specified vision
and mission?
The firm's vision is a picture of what it wants to be and what it wants to ultimately achieve. The firm's
mission is based on its vision. It specifies the business(es) in which the firm intends to compete and the
customers it intends to serve. The value of having a vision and mission is that they inform stakeholders what the
firm is, what it seeks to accomplish, and who it seeks to serve. A successful vision is inspirational. The mission
is more concrete and guides employees' behavior as they achieve the firm's vision. Research shows that an
effectively formed vision and mission positively impact firm performance in terms of growth in sales, profits,
employment, and net worth.
6. Describe an organization's various stakeholders and their different interests. Under what
condition can the firm most easily satisfy all stakeholders? If the firm cannot satisfy all stakeholders,
which ones must it satisfy in order to survive?
Stakeholders are the individuals and groups who can affect and are affected by the strategic outcomes
achieved and who have enforceable claims on a firm's performance. There are 3 principal types of stakeholders:
1) Capital market stakeholders: the shareholders and the major suppliers of capital to the firm. They are
most interested in the return on capital in relation to the risk incurred.
2) Product market stakeholders: customers, suppliers, host communities, and unions representing workers.
The customers seek a reliable product at the lowest possible price. The suppliers seek loyal customers willing to
pay the highest sustainable price. Host communities want companies willing to be long-term employers and
providers of tax revenues. Union officials want secure jobs with good working conditions for the workers they
represent.
3) Organizational stakeholders: employees (managerial and non-managerial). These stakeholders expect a
firm to provide a dynamic, stimulating, and rewarding work environment. The firm can most easily satisfy all
stakeholders if it earns above average returns. If the firm does not earn above-average returns, it must prioritize
its stakeholders by their power, urgency, and degree of importance to the firm. The firm must then make trade-
offs among the stakeholders.
7. Who are the firm's strategic leaders? How do strategic leaders predict the profit outcomes of
different strategic decisions?
The firm's strategic leaders include the CEO and top-level managers, but they also include organizational
members who have been delegated strategic responsibilities. Strategic leaders use the strategic management
process to help the firm reach its vision and mission. Mapping an industry's profit pool is one way strategic
leaders can anticipate the profitability of different strategic decisions. A profit pool is the total profits earned in
an industry along all points in the value chain. This helps the leaders determine where the primary sources of
profit in the industry are located and allows them to take actions to tap these sources.
8. Explain the relationship of the strategic management process to organizational ethics.
Almost all strategic management process decisions have ethical implications because they affect
stakeholders. The decision of the strategic leaders influence the organization's culture which is based on the
organization's core values (which are also influenced by the strategic leaders). The organization's culture can be
functional or dysfunctional, ethical or unethical. Consequently, the strategic leader's role has a large impact on
whether the organization is a good citizen.
9. What are the primary aspects of the strategic management process? You may reference specific
chapters from the text in formulating your response.
The strategic management process consists of three primary processes:
1) Analysis: involves the development of an understanding of the external environment and internal
organization
2) Strategy formulation: with knowledge about its external environment and internal organization, the firm
forms its vision and mission and makes decisions as to what strategies to utilize to provide returns to
shareholders
3) Implementation: putting the formulated plan into action
10.Explain why it is important for organizations to analyze and understand the external
environment.
Organizations do not exist in isolation. The external environment of the organization presents threats and
opportunities which the organization must address in its strategic actions. Parts of the organization's external
environment are changing rapidly, such as technology, and the organization must constantly adjust to these
changes. The information that the organization gathers about competitors, customers and stakeholders is used to
build the organization's capabilities or to build relationships with stakeholders in the external environment. The
information that the organization gathers about the external environment must be matched with its knowledge of
its internal environment to form vision, to develop its mission, and to take actions that result in strategic
competitiveness and above-average returns.
Case Scenario 1: Palmetto. Palmetto was an early pioneer of personal data assistants (PDAs) and
dominates that market space (in terms of market share) with its core product, the Palmetto Pidgy.
Because this product category was entirely new to the market, Palmetto had to internally develop the
hardware and software sides of the business, and today is both a manufacturer of PDAs and a
programmer and licensor of its PDA operating system software. Recently, however, the hand-held device
maker's performance has taken a dive as a result of slumping sales and costly inventory problems. New
large entrants are entering both the equipment and software sides of its business, putting further
pressure on margins. Management is currently considering its options, including the break up of
Palmetto into two separate, independent public companies - one devoted to hardware, the other software.
-(Refer to Case Scenario
1) What primary business strategy issues does Palmetto face?
Recognizing that students have only just been introduced to strategy in this introductory chapter, the
Palmetto scenario helps frame and contrast the basic business and corporate strategy questions. The best
answers to the first question will start by noting that Palmetto appears to be in two distinct businesses, hardware
and software, which in turn are likely to have very different success factors and competitors. Students can then
begin talking about these competitors and the potential resources they bring to the table (for instance, Microsoft
in software and Sony in miniaturized consumer electronics). This scenario also leads to a natural discussion of
the attractiveness of the PDA market, and where the most money is likely to be made.
2) What primary corporate strategy issues does Palmetto face?
Since the business strategy question should have revealed that Palmetto is actually in at least two distinct
businesses, the best answers to the corporate strategy question will begin by assessing which of the businesses is
more attractive, and whether or not Palmetto needs to be in both to compete, or should specialize in either
software or hardware. Companies which are diversified will have a corporate strategy that encompasses various
businesses with different business strategies. Students can be prompted to debate the tradeoffs between retaining
both businesses versus breaking the company in two -a useful role play exercise entails asking students to walk
through the likely resource allocation tradeoffs that the diversified Palmetto must currently make
3) How do the I/O and resource-based models help you make recommendations to Palmetto's
management regarding a split into two companies? Do they lead to the same recommendation?
The best answers will begin by noting that the two models should be viewed as complementary and applied
in an integrative manner. Since the perspectives are complementary, the choice of I/O or resource-based
perspective as a starting point is simply a matter of taste. For instance, the discussion can then flow to how the
I/O perspective will help management understand the characteristics of the two basic industries in which it
participates (hardware and software), and perhaps lead to insights into what factors allow one firm to compete
effectively against other industry incumbents. The resource-based model can then be applied to develop an
understanding of where Palmetto is strongest in terms of resources, capabilities, and core competencies. Further
industry analysis can show whether or not these resources will likely lead to competitive advantage in their
respective markets. Through the combination of these two perspectives, students can then help management
determine whether Palmetto can afford to remain a diversified firm or if it can only compete effectively by
focusing on either its hardware or software business
Case Scenario 2: Jewell Company. Jewell Company is a diversified manufacturer and marketer of
simple household items, cookware, and hardware. In its annual report, it expresses its strategy as follows:
"Jewell manufactures and markets staple volume lines to the volume purchaser. We aim to increase
shareholder value by continuing to build a company with superior earnings per share growth and return
on investment (ROI), and to earn a reputation for excellence in performance and management. We plan
to do this by merchandising to the customer goods market a multi-product offering with superior
customer service performance for maximum market leverage. Through this we will achieve an ROI of
20% plus EPS growth of 15%, with the constraint that debt not exceed half of our equity." -(Refer to
Case Scenario 2)
1) What are the strengths and weaknesses of the above strategy statement?
Good strategies help managers to make tough decisions, which necessarily require them to
make tradeoffs. In terms of strengths, the best answers will note that Jewell’s strategy is fairly precise with
regards to target customers (volume purchasers like Walmart), product characteristics (staples, manufactured
items),how the firm plans to win the customers’ business (merchandising and multiple-product offering), and
measurable performance objectives. In terms of weaknesses, staple manufactured products encompass a
multitude of potential products (pens to hairbrushes to curtain rods) so the statement does not signal any
particular type of manufacturing expertise. The statement also does not tellus how the firm plans to
grow (internal means versus acquisition). Finally,management has stated a growth goal, not a growth
plan. Absent a clear plan,strategic compromises and inconsistencies in the pursuit of growth will likely
erode the competitive advantage that Jewell had with its original varieties of product offerings and
target customers.
2) Which groups of stakeholders does Jewell's statement appear to speak to?
This statement focuses on capital market stakeholder groups and one product market group, the customers.
This is because Jewel manufactures and markets staple volume lines to the volume increase. They want their
product get high demand from customer as well as customer has many products to choose from then get the
increasing in ROI. Customers are become their focuses to speak to and then maximum their market leverage.
3) Does Jewell Company's statement of strategy include a vision statement or a mission statement?
Why or why not?
Strategic intent is internally focused. It is concerned with identifying the resources,
capabilities, and core competencies on which a firm can base its strategic actions. While strategic
mission is a statement of a firm's unique purpose and the scope of its operations in product and market
terms. Strategic Mission flows from strategic intent. It is externally focused.It describe strategic intent and
strategic mission when Jewell mention that they want to aim to increase shareholder value by continuing to
build a company with superior earnings per share growth and ROI, and to earn reputation for
excellence in performance and management. Besides that Jewell also said that they plan to do the merchandising
to the customer good market a multi-product offering with superior customer service performance for
maximum marketleverage.
Case Scenario 3: Vivendi Universal SA. Vivendi Universal is a French firm that started in 1853 as
Companie General des Eaux. It grew from a French water utility company into one of the world's largest
conglomerates. Under the corporate leadership of CEO Jean Marie Messier, Vivendi Universal became a
highly diversified company involved in music, publishing, film, pay TV, telecoms, Internet, water
distribution, thermal energy supply, building and heavy public construction projects, waste management,
electrical energy services, real estate and other activities. Mr. Messier was forced out of the company in
July, 2002, in a liquidity crisis and mounting shareholder anger. The acquisitions made by Mr. Messier
saddled the company with billions of dollars of debts. Vivendi shares plummeted 80 percent during the
last six months Mr. Messier was CEO, according to the Wall Street Journal. Meanwhile, the SEC
indicated that a disputed severance payment of $23 million to Mr. Messier may actually constitute "ill
gotten gains," reported the Wall Street Journal. Vivendi Universal refused to make the payment saying
the board and shareholders had never agreed to the severance payment. On the brink of bankruptcy,
Vivendi Universal brought in Jean-Rene Fourtou to replace Mr. Messier as CEO. According to the
business media, Mr. Fourtou has taken a dying enterprise and given it a survival plan. He sold numerous
Vivendi Universal businesses, bringing the company to focus on Cegetel, a phone company; SFR, a
cellphone company; Canal Plus, a television company; and Universal Music. Mr. Fourtou was able to
reduce Vivendi's debt from 37 billion euros in 2002 to a projected 5 billion euros by the end of 2005. The
company showed its first quarterly profit at the end of 2003, allowing Mr. Fourtou to arrange a loan from
a banking consortium and give the company hopes that credit-rating agencies would raise its debt from
junk-bond status, according to The New York Times. The company projects $2.2 billion in profits for
2005. According to Barrons, "Fourtou and the new chief executive Jean Bernard Levey have moved
beyond restructuring and recapitalizing the firm to building core businesses." -(Refer to Case Scenario 3)
1) Who are stakeholders of Vivendi Universal, and what was the role of stakeholders in Vivendi
Universal's recent history?
Stakeholders are the individuals and groups who can affect, and are affected by, the strategic outcomes
achieved by the firm and who have enforceable claims on a firm’s performance. Stakeholders support an
organization as long as its performance meets or exceeds their expectations. As Vivendi Universal grew ever
larger and more diverse, its financial performance declined. Thus it lost the support of its capital market
stakeholders, its shareholders. They rebelled, and the result was the firing of Mr. Messier, the instigator of the
growth, and the installation of Mr. Fourtou, who immediately began divesting most of the companies Mr.
Messier had purchased. A second stakeholder group was the individuals and organizations holding Vivendi
Universal debt (bonds and bank loans) that were threatened by the impending bankruptcy of the firm. Mr.
Fourtou pleased this group of stakeholders by reducing the debt of the firm by selling off the excess companies.
Finally, shareholders were also supported by the SEC, which is investigating the multi-million severance
payment to Mr. Messier.
2) Who was ultimately responsible for the problems at Vivendi Universal?
Some believe that every organizational failure is actually a failure of those who hold the final responsibility
for the quality and effectiveness of a firm’s decisions and actions. Strategic leaders are the people responsible
for the design and execution of strategic management processes. At Vivendi Universal, Mr. Messier, the former
CEO, seems to have borne the brunt of public blame. But Vivendi’s top management team and the board of
directors must assume some of the blame because Mr. Messier did not act alone. The pivotal role that can be
played by a CEO as a strategic leader is also illustrated by the successful changes instituted by Mr. Fourtou, the
new CEO.
Quiz 2: The External Environment
https://wenku.baidu.com/view/c48d2116f18583d0496459c7.html
https://quizlet.com/28178805/strategic-management-case-scenario-application-questions-flash-
cards/
https://testbankplus.com/sample/pdf/9781285425177-TEST-BANK.pdf
1. Identify and describe the three major parts of the external environment. What is the purpose of
the firm's collecting information about these aspects of its environment?
The external environment has three major parts. The first is the general environment, which is composed of
dimensions in the broader society that affect industries and their firms. These environmental segments are:
demographic, economic, political/legal, sociocultural, technological, and global. The second part of the external
environment is the industry environment, which involves five factors that influence a firm, its competitive
actions and responses, and the industry's profit potential. These five factors are: the threat of new entrants, the
power of suppliers, the power of buyers, the threat of product substitutes, and the intensity of rivalry among
competitors. The competitor environment is the third part of the external environment. The firm must be able to
predict competitors' actions, responses, and intentions. With the information collected about these aspects of its
external environment, the firm can develop its vision, mission, and strategic actions.
2. Describe and discuss the four activities of the external environmental analysis process.
The external environmental analysis process includes four steps: scanning, monitoring, forecasting, and
assessing. The scanning of the environment includes the study of all segments of the general environment in
order to detect changes that may occur in the future or that already are occurring. This is critical in a volatile
environment. Scanning often deals with ambiguous, incomplete, or unconnected data and information. When
analysts monitor the environment, they observe environmental changes to see if an important trend is emerging
from those spotted by scanning. It is critical for the firm to detect meanings in these events and trends so that it
can be prepared to take advantage of opportunities these trends provide. Forecasting builds on scanning and
monitoring to develop feasible projections of what might happen and how quickly it will occur. Forecasting is
important in helping the firm adjust sales to meet demand. Finally, through assessing, the analyst determines the
timing and the significance of the effects of environmental changes and trends on the strategic management of
the firm. Assessment must specify the competitive relevance of the data
3. Describe the seven segments of the general environment.
1) The demographic segment encompasses factors such as population size, geographic distribution, age
structure, ethnic mix, and income distribution.
2) The economic segment involves the nature and direction of the economy in which a firm competes or
may compete, domestic as well as global.
3) The political/legal segment is the arena in which organizations compete for attention, resources, and a
voice in laws and regulations guiding the interactions among nations.
4) The sociocultural segment is concerned with society's attitudes and cultural values.
5) The technological segment includes institutions and activities involved with creating new knowledge
and transforming it into new outputs, products, processes, and materials.
6) The global segment includes new global markets, existing markets that are changing, international
political events, and critical cultural and institutional characteristics of global markets.
7) The physical segment includes potential and actual changes in the physical environment (such as global
warming) and business practices that are intended to positively deal with those changes (such as control of
carbon emissions and other environmentally friendly actions).
4. What are high exit barriers and how do they affect the competition within an industry?
Exit barriers are economic, strategic, and emotional factors causing companies to remain in an industry,
even though the profitability of doing so is in question. The following are common sources of exit barriers: 1)
specialized assets which cannot be used in another business or location; 2) fixed costs of exit, such as labor
agreements which penalize a firm for ceasing operation; 3) strategic interrelationships or mutual dependence of
business units wherein one business of a corporation serves another corporate business; 4) emotional barriers
that cause owners to be sentimentally attached to the business or to their own role in it; 5) governmental and
social restrictions that prevent a firm from closing, often in order to prevent the loss of jobs in a country or
community. \
5. Identify the five forces that underlie the five forces model of competition. Explain briefly how
they affect industry profit potential.
1) Threat of new entrants: New entrants threaten existing firms' market share. They increase production
capacity in an industry which results in lower profits for all firms, unless demand is increasing. The new entrant
may force the existing firms to be more effective and efficient in production, and to compete on new
dimensions.
2) Power of suppliers: Suppliers with high power can increase prices and decrease the quality of their
products sold to the firm. If firms are unable to pass along price increases to customers, their profits diminish.
3) Power of buyers: When buyers (customers) have high power they can force prices down, and require
increases in quality and service
levels, thus driving profits down.
4) Substitutes: Substitutes perform the same or similar functions of the firm's product. The price of the
substitute places an upper limit on prices firms can charge for the original product, limiting industry profits.
5) Intensity of competitive rivalry affects the firm's ability to make a profit as competitors' actions
challenge the firm or competitors try to improve their market position. Increasing rivalry reduces the ability of
weaker firms to survive.
6. What is a firm's strategic group? What effect does the strategic group have on the firm?
The firm's strategic group is the set of firms that emphasize similar strategic dimensions and use a similar
strategy. The firms in a strategic group occupy similar positions in the market, offer similar goods to similar
customers, and may make similar decisions about production technology and organizational features.
Competition among firms in a strategic group is more intense than the competition among a firm and those
firms outside its strategic group. Actions of members in the firm's strategic group affect its strategic decisions in
many areas including pricing, product quality, and distribution.
7. What do firms need to know about their competitors? What legal and ethical intelligence
gathering techniques can be used to obtain this information?
Competitor analysis helps firms identify:
1) what drives the competitors by understanding the competitor's FUTURE objectives);
2) what the competitor is doing and is capable of doing by understanding the competitor's CURRENT
STRATEGY;
3) what the competitor believes about the industry by understanding the ASSUMPTIONS made by the
competitor; and
4) what the competitor's CAPABILITIES are by understanding the competitor's strengths and weaknesses.
Firms can legally and ethically gather public information, such as annual reports, SEC reports, UCC filings,
court records, and advertisements. Firms can also attend trade fairs to obtain competitors' brochures, view
exhibits, and discuss products. This data combines to form competitive intelligence.
8. What are the barriers to entry and how do they affect competition?
Entry barriers discourage competitors from entering a market and facilitate a firm's ability to remain
competitive in a market in which it currently competes. Barriers to entry include:
1) Economies of scale are derived from incremental efficiency improvements through experience as a firm
grows larger.
2) Product differentiation occurs when over time, customers may come to believe that a firm's product is
unique. This belief can result from the firm's service to the customer, effective advertising campaigns, or being
the first to market a product or service.
3) Capital requirements – Competing in a new industry requires a firm to have resources to invest. In
addition to physical facilities, capital is needed for inventories, marketing activities, and other critical business
functions.
4) Switching costs are the one-time costs customers incur when they buy from a different supplier.
5) Access to distribution channels – Over time, industry participants commonly learn how to effectively
distribute their products. Once a relationship with its distributors has been built a firm will nurture it, thus
creating switching costs for the distributors.
6) Cost disadvantages independent of scale – Sometimes, established competitors have cost advantages that
new entrants cannot duplicate. Proprietary product technology, favorable access to raw materials, desirable
locations, and government subsidies are examples.
7) Government policy – Through their decisions about issues such as the granting of licenses and permits,
governments can also control entry into an industry.
Case Scenario 1: The Boys and Girls Club. The Boys and Girls Club (BGC) is a national non-profit
organization geared to provide America's youth with the tools and skills they need to become healthy
adults, responsible citizens, and effective leaders. By bringing parents, neighbors, educators, and civic
leaders together with our youth, BGC believes it can instill these crucial life lessons at an age when
they're most needed. The national organization is headquartered in Atlanta, GA, and serves as a service
hub for over 3,700 club locations around the U.S. Each local club is directed by a volunteer board of
directors and staffed by professional youth development workers (usually including an executive director,
a program director, and an arts director) and many volunteers who just enjoy working with young people
and want to make a difference in their lives. While affiliated with the national center, each local BGC is
locally funded. -(Refer to Case Scenario 1)
1. How are the various facets of the general environment (Table 2.1 in Strategic Management) likely
to be important for BGC?
The best answers will begin by noting that BGC has a mission focused on the education and social
development of needy youth. Thus, the demographic, economic, sociocultural. and physical segments may be
the segments of primary importance. Within the physical segment, for instance, BGC may consider what it can
do to respond to climate change and depletion of energy resources. The global segment is also a natural
discussion point since contexts far from home may not come to our attention until after a critical stage has been
passed. For instance, the presence of immigrants and refugees in a community affect the needs of BGC's
clientele.
2. Why would attention focused on victims of natural disasters be a threat to the BGC?
The best answers will observe that BGC is entirely dependent upon local donations for its operations and
public focus on other causes will likely draw away donation dollars that had been historically earmarked for
BGC. This alternative charitable giving serves donors as a substitute for donations to BGC.
3. How might the BGC respond to threats to their donations at both local and national levels?
Since BGC is governed locally by a board of directors drawn from the community, the local organizations
should use these members to rally support against their dwindling donation base. The board and BGC staff
members can also reach out to other local organizations and community govermments. At a national level,
image ads and the lobbying of various national organizations (government, teachers' associations, minority
outreach organizations, environmental groups, etc.) can be initiated and managed through the BGC headquarters
in Atlanta.
1. Get more media support (newspapers, magazine)
2. Provide testimony and evidence that project is working
3. Competition - to create awareness
4. Centralization of national entity for donation
Case Scenario 2: B.B. Mangler. B.B. Mangler is a top U.S. business-to-business distributor of
maintenance, repair, and service equipment, components, and supplies such as compressors, motors,
signs, lighting and welding equipment, and hand and power tools. Its industry is typically referred to as
MRO, which is an acronym for maintenance, repair, and supplies. MRO products are typically small,
fairly inexpensive (light bulbs and washers), but often needed on short notice. It states its strategy as
having the "capacity to offer an unmatched breadth of lowest total cost MRO solutions to business."
Mangler's GoMRO sourcing center for indirect spot buys locates products through its database of 8,000
suppliers and 5 million products. Mangler has 388 physical branches in the U.S., including Puerto Rico
(90% of sales), 184 in Canada, and 5 in Mexico. Customers include contractors, service and maintenance
shops, manufacturers, hotels, governments, and health care and educational facilities. Mangler also
provides materials-management consulting services. -(Refer to Case Scenario 2)
1. Historically, Mangler appears to have relied on its physical locations for market presence in the
U.S. and northern South America. What threats does the Internet pose to its location-based strategy?
The best answers will start by noting that Mangler's location-based strategy is also likely to require quite a
bit of investment in inventory (keeping all those parts on hand at each of its branches in the United States,
Canada, and Mexico). Given that it competes in a low-cost industry. and itself competes on cost. an Internet-
based MRO competitor may be able to create an even lower cost structure (as Amazon.com did with books).
The Internet seems like a natural fit for the MRO market. Such an online strategy may be particularly effective
for those MRO items that are less time-critical.
2. What opportunities does the Internet provide to Mangler, both domestically and internationally?
Answers to this question should suggest several different responses to the ways in which Mangler could
capitalize on the Internet domestically. The best answers for the international strategy question will begin by
noting that just as Mangler's many domestic locations provide a barrier to entry in its markets by potential
competitors (that is, it already has the market share to cover its high physical location costs and also is likely to
have tremendous goodwill). so too have they been a barrier against Mangler's entry into other international
markets such as Europe, Asia, and other parts of Latin America. The Internet does away with this barrier to a
great extent, which levels the playing field between Mangler and the incumbents of those respective
international markets.
3. How should Mangler respond to the threat of new Internet-based entrants?
There are several possible avenues, and the best answers will note these alternatives. The most obvious
response would be for Mangler to start up a web-based complement to its location-based delivery system. A
related response might involve the centralization of low-demand, high-cost items to specific areas of the
country, where they could then be funneled rapidly to the actual local outlets using the Internet as an internal
market. Finally, Mangler could hedge this threat by investing in the most promising online rivals.
Case Scenario 3: Barracuda Inc. Barracuda Inc. is a lamp fixture manufacturer that is considering
an entry strategy into the U.S. home furnishings manufacturing industry. The existing landscape consists
of many players but none with a controlling share. There are presently 2500 home furnishings firms, and
only 600 of those have over 15 employees. Average net profit after tax is between 4 and 5%. While the
industry is still primarily comprised of single-business family-run firms that manufacture furniture
domestically, imports are increasing at a fairly rapid rate. Some of the European imports are leaders in
contemporary design. Relatively large established firms are also diversifying into the home furnishings
industry via acquisition. Supplier firms to the home furnishings industry are in relatively concentrated
industries (like lumber, steel, and textiles). Retailers, the intermediate customer of the home furnishings
industry, have been traditionally very fragmented. Customers have many products to choose from, at
many different price points, and few home furnishing products have strong brands. Also, customers can
switch easily among high and low-priced furniture and other discretionary expenditures (spanning big
screen TVs to the choice of postponing any furniture purchase entirely). -(Refer to Case Scenario 3)
1. Using the five-forces framework, summarize the opportunities and threats facing Barracuda as it
considers entry into the home furnishings manufacturing industry. Which threats are greatest to current
incumbents?
The best answers will be based on an application of the five forces model to the scenario, From this model
students should be able to point out that the most significant threats are the power of consumers, lack of
economic power with suppliers, and increasing presence of imports. These characteristics plus the highly
fragmented nature of the industry itself are likely to translate into near-perfect competition, leaving no single
player with a clear advantage. Opportunities may exist in particular niches, depending on the internal strengths
of new entrants. In terms of the larger market, there appears to be an opportunity for a large firm to consolidate
the industry and add brand power, thereby potentially gaining power over suppliers and customers.
2. How intense is competitive rivalry likely to be among incumbents of the home furnishings
manufacturing industry?
The best answers will be able to walk through the determinants of rivalry spelled out in pages 57 through
59. The fact that this industry is fairly characterized as having nearly perfect competition suggests that rivalry is
high. Larger players are likely to have significant exit barriers. particularly given the slow growth. high fixed
costs, lack of differentiation, and low profitability of the market overall. Thus, new larger entrants to this
industry may further escalate the degree of competition.
3. Is the furniture industry described above attractive?
Astute students may begin by noting that this industry is attractive if you are in a position that is currently
less attractive than that demonstrated by the home-furnishings business. Beyond that, discussion should
generally lead to the recognition that this industry is currently unattractive - summarized by its paltry profit
margins, fragmented membership. lack of power over suppliers and customers, and high degree of rivalry.
Quiz 3: The Internal Organization
https://wenku.baidu.com/view/9f32b80503d8ce2f006623c7.html
1. Describe the importance of internal analysis to the strategic success of the firm.
By analyzing its internal environment, a firm determines what actions it can take based on its unique
resources, capabilities and core competencies. The firm's core competencies are the source of the firm's
competitive advantage. Internal analysis allows the firm to compare what it is capable of doing (what it "can
do") with what it "might do" (which is a function of opportunities and threats in the external environment).
Matching what a firm can do with what it might do allows the firm to develop its vision, pursue its strategic
mission, and select and implement its strategies. This allows the firm to leverage its unique bundle of resources
and capabilities to gain competitive advantage.
2. What are the differences between tangible and intangible resources? Which category of resources
is more valuable to the firm?
Resources are either tangible or intangible. Tangible resources are those assets that can be observed and
quantified. There are 4 types of tangible assets:
1) financial resources (borrowing capacity, ability to generate internal funds);
2) physical resources (plant and equipment, access to raw materials);
3) technological resources (patents, trademarks, copyrights, and trade secrets); 4) organizational resources
(formal reporting structure, planning, controlling and coordinating systems).
Intangible resources are those assets in the firm that are less visible. There are 3 types of such resources:
1) human resources (knowledge, trust, management capabilities, and organizational routines),
2) resources for innovation (ideas, scientific capability, and capacity for innovation), and
3) reputation (reputation with customers, i.e., the firm's brand name and perceptions of product quality, and
relationships with suppliers). Intangible assets develop over time and are deeply rooted in the organization's
history. Consequently, they are difficult for competitors to analyze and imitate. In addition, intangible resources
can be leveraged to create new value to the firm.
These properties give intangible resources a greater ability to create sustainable competitive advantage than
do tangible resources.
3. Define capabilities and how they affect the firm's strategic success.
Capabilities exist when resources have been purposely integrated to achieve a specific task or tasks.
Examples: HR activities, product marketing, R&D.
Capabilities are based on developing, carrying, and exchanging information and knowledge through the
firm's human capital. Many of the firm's capabilities are based on the unique skills and knowledge of its
employees and their functional expertise. The knowledge possessed by human capital is among the most
significant of a firm's capabilities. Capabilities are often developed in specific functional areas or in part of a
functional area.
4. Describe the four specific criteria that managers can use to decide which of their firm's
capabilities have the potential to create a sustainable competitive advantage.
Managers must identify whether their firm has capabilities that are valuable and nonsubstitutable from the
customer's point of view, and unique and inimitable from the firm's competitors' point of view. Only capabilities
with these four characteristics are core competencies that can lead to sustainable competitive advantage:
1) Valuable
2) Rare
3) Costly-to-imitate
4) non-substitutable
5. Describe a value chain analysis. How does a value chain analysis help a firm gain competitive
advantage?
A value chain analysis allows a firm to understand the activities that create value for the firm and those that
do not. A value chain follows the product from its raw-material stage to the final customer. The purpose is to
add as much value as possible as cheaply as possible and to capture that value. To conduct a value chain
analysis, managers should study and identify all activities of the firm and evaluate their impact on the effort to
create value for the customer. Two central activities in a value chain:
1) Primary activities: involved in a product's physical creation, its sale and distribution, and its service after
the sale
2) Support activities: necessary for the primary activities to take place
6. Why is it important to prevent core competencies from becoming core rigidities?
All core competencies have the potential to become core rigidities and to generate failure. Each
competence is a potential weakness if it is emphasized when it is no longer competitively relevant. The success
that the competence generated in the past can generate organizational inertia and complacency. A core
competence can become obsolete if competitors figure out a better way to serve the firm's customers, if new
technologies emerge, or if political or social events shift in the external environment. If the organization's
managers react to these changes with inflexibility and strategic myopia, then core rigidities are created.
7. What is value? Why is it important?
Value measured by a product's performance characteristics and by its attributes for which customers are
willing to pay.
8. Define outsourcing. Why do organizations outsource?
The purchase of a value-creating activity or a support function activity from an external supplier.
9. Why is it important to identify internal strengths and weaknesses?
10.Describe an organization with which you are familiar. Does it have a sustainable competitive
advantage?
Case Scenario 1: Heartsong LLC. Heartsong LLC is a designer and manufacturer of replacement
heart valves based in Peoria, Illinois. While a relatively small company in the medical devices field, it has
established a worldwide reputation as the provider of choice high-quality, leading-edge artificial heart
valves. Most of its products are sold to large regional hospital systems and research hospitals. Specialty
heart centers are another emerging, but fast-growing, market for its valves. While Heartsong would like
to grow quickly, its growth is constrained by the need to finance larger production runs and then carry
this additional inventory. For products like those of Heartsong, vendors typically do not collect payment
until the unit is actually used in surgery. Moreover, heart valves are usually required on short notice
which means that they must be either onsite, or inventoried at a nearby location. If nearby, then
transport of the unit to a hospital or heart center occurs within a matter of hours, and sometimes
minutes. For this reason, accelerated growth would require Heartsong to both finance increased
production of its heart valves, along with carrying increased levels of inventory that are in fact sitting on
their customers' shelves. In fact, inventory-carrying cost is its single largest cost outside of research and
development. While profitable growth is necessary if Heartsong is to continue extending its competitive
advantage through increasingly greater investments in basic heart valve R&D, it is not clear that the
company can internally support all these increased financial commitments (R&D, manufacturing, and
inventory). Doc Watson, the CEO of Heartsong, is considering an outside contractor, EdFex, to handle
the inventorying, warehousing, and delivery of its valves. EdFex has secure, high-tech warehouses in most
major population centers around the country, and can ensure delivery of a product to these markets from
its warehouses in less than one hour. -(Refer to Case Scenario 1)
1. What value-chain activities appear to underlie Heartsong's competitive advantage?
The best answers will begin by noting that Heartsong has the capacity to design leading-edge medical
products and then take these designs and turn them into reliably manufactured, high-quality replacement heart
valves. Thus, basic R&D and quality precision manufacturing are likely to be critical value-creating facets for
this firm
2. Why might an outsourcing arrangement with EdFex be attractive to Heartsong?
The best answers will start by observing that the scenario suggests that Heartsong needs to grow if it is
going to continue being competitive and successful. However, Heartsong is also capital constrained and an
outsourcing arrangement with EdFex allows it to more efficiently manage this significant aspect of its cost base
(inventory and delivery). This outsourcing solution would be ideal if it would allow Heartsong to maintain a
centralized warehouse with heart valve inventory in major population centers, instead of its present practice of
carrying inventory on the shelves of each of its hospital customers. As a result, Heartsong could grow its market
presence, while more efficiently managing the need to have heart valves available on short notice.
3. What are the implications of an EdFex outsourcing arrangement for the capabilities underlying
Heartsong's competitive advantage?
The best answers will develop the theme that the EdFex outsourcing arrangement is truly likely to be win-
win. With the arrangement in place, Heartsong is able to devote its financial, human capital, and managerial
resources to basic R&D and quality precision manufacturing; and, EdFex does what it does best in logistics.
Moreover, it is hard to contemplate that EdFex would ever think of entering the heart valve industry - thus,
EdFex does not pose a direct threat as a future competitor. It does however pose an indirect threat to Heartsong
to the extent it can hold the firm hostage, and extract exorbitant fees for its logistic services.
Case Scenario 2: ERP Inc. ERPI is a leading provider of enterprise integration software (EIS). EIS
allows a firm to connect and integrate processes across all aspects of its business, regardless of where they
are located around the world. ERPI is a product-focused company, whereas most competitors in its
market space, like Oracle, operate as "solutions companies." Oracle and Microsoft have begun to devote
considerable resources to the development of and acquisition of products to compete in the EIS space.
Despite these recent threats, one benefit of its product-focused strategy is that ERPI's proprietary
product is generally recognized as being 200% to 300% better than competitors' software. ERPI
estimates it will take 2 to 3 years for competitors to develop the capabilities needed to bring a competing
product to market. ERPI invests a considerable percentage of its profits in basic R&D to support its core
products. As evidence of this, among its competitors the firm maintains the largest in-house
programming staff dedicated solely to the development of advanced enterprise integration software.
Installation and related consulting for EIS typically cost between $100 and $200 million, with the ERPI
software component accounting for about 20% of the installed cost (the remaining 80% is spent on the
actual installation, not counting the value of the customer's time). ERPI's target market consists of the
world's largest manufacturing and industrial firms and it currently enjoys a 60 percent market share. -
(Refer to Case Scenario 2)
1. How valuable, rare, costly to imitate, and non substitutable are ERPI's capabilities?
The best answers will simply walk through the respective columns in Table 3.5 and reach the conclusion
that, at least in the near term, ERPI has a sustainable competitive advantage. Its EIS software is valuable given
that it is 200% to 300% better than competitors’ products. It is similarly rare and nonsubstitutable since it is
proprietary, and currently has a two-year lead on the alternatives. A similar rationale can be invoked to support
the argument that ERPI’s capabilities in software programming are going to be costly to imitate. A competitor
would have to hire a similar workforce or acquire a company that currently occupies the same market space.
This strong position is further bolstered by the fact that a large percentage of the market is voting with its feet in
favor of ERPI.
2. How sustainable is ERPI's competitive advantage?
The best answers will build on the basic notions developed in response to question 4. Students will argue
that ERPI’s competitive advantage is sustainable as long as its technology continues to define the leading edge
of EIS products and that substitute solutions do not encroach much on its two-year lead. However, and as is
consistent with most high-technology markets, as students pick apart ERPI’s capabilities following the
categories in Table 3.5 they should begin to see that sustained competitive advantage in this particular market
space may be difficult, particularly given the presence of large, aggressive competitors like Oracle and
Microsoft, which are intent on gaining a presence in the EIS market.
3. Imagine that ERPI's historic growth strategy has focused on making one sale and then moving on
to the next target company. After several years of building market share using this approach, what new
resources has ERPI developed?
This question asks students to take a more dynamic perspective of potentially valuable resources that
companies and their customers create together, but that the company itself can exploit (a perfect example of a
co-specialized asset). The best answers will begin by observing that if ERPI has focused historically on
transactions (making the sale), then it has given little explicit consideration to customers as long-term
relationships beyond the need to provide technical support (lifetime value of a customer beyond the first sale).
Shifting attention to ERPI installations as relationships suggests that the company now has a customer list to die
for. This list is especially valuable since (1) the target companies have invested upwards of $200 million in
ERPI proprietary systems and, (2) once installed, given the pervasive nature of EIS systems, those target firms
are unlikely to simply switch to another system.
Quiz 4: Business-Level Strategy
https://quizlet.com/272280883/mgmt-4900-midterm-flash-cards/
https://www.studyblue.com/notes/note/n/chapter-4-business-level-strategy/deck/7118779
https://tbmirate.com/product/strategic-management-concepts-and-cases-competitiveness-
and-globalization-11th-edition-by-michael-a-hitt-test-bank/
1. Define strategy and business-level strategy. What is the difference between these two concepts?
In general, a strategy consists of the choices an organization makes in an attempt to gain strategic
competitiveness and earn above-average returns. The organization's strategic choices are influenced by threats
and opportunities in the external environment and by the nation and quality of its internal re-sources,
capabilities, and core competencies. The strategy reflects the firm’s vision and mission. Business-level strategy
is concerned with a particular product market. Business-level strategy is an integrated and coordinated set of
commitments and actions the firm uses to gain a competitive advantage in a particular product market. It is the
organization's core strategy. Every firm, no matter how small, will have at least one business-level strategy. A
diversified firm will have several types of corporate-level strategies as well as a separate business-level strategy
in each product market area in which the company competes. The essence of a firm’s business level strategy is
choosing to perform activities differently or to perform different activities than competitors.
2. When a firm chooses a business-level strategy, it must answer the questions "Who? What? and
How?" What are these questions and why are they important?
- Who: Determining the customers to serve. It is important to know your market segments. This allows you
to tailor your strategy to their specific needs
- What: Determining which customer needs to satisfy. Customer needs are related to a product's benefits
and features. Fulfilling customer needs is how firms create value.
- How: Determining core competencies necessary to satisfy customer needs. Only firms with capacity to
continuously improve, innovate and upgrade their competencies can expect to meet and/or exceed customer
expectations across time.
The firm must decide (1) who are the customers who will be served, (2) what needs do the target customers
have that must be satisfied, and (3) how will those needs be satisfied by the firm. The choice of target customer
(who) usually involves segmenting the market to cluster people with similar needs into groups. The target
customers’ needs drive “what” benefits and features the firm’s product will have. This involves a choice and
balance between cost and differentiation of the product. Finally, firms use their core competencies (how) to
implement value-creating strategies and satisfy customers’ needs.
3. Discuss how a cost leadership strategy can allow a firm to earn above average returns in spite of
strong competitive forces. Address each of the five competitive forces.
A firm focuses on a niche market, adding value by leveraging value chain activities that allow value
creation through the cost leadership strategy.
• Competitive advantage: low cost
• Competitive scope: narrow industry segment
Rivalry: Having the low-cost position serves as a valuable defense against rivals. Because of the cost
leader's advantageous position, especially in logistics, rivals cannot reduce their costs lower than the cost
leaders', and so they cannot earn above-average returns.
Buyers: The cost leadership strategy also protects against the power of customers.Powerful customers can
drive prices lower, but they are not likely to be driven below that of the next-most-efficient industry competitor.
Prices below this would cause the next-most-efficient competitor to leave the market, leaving the cost leader in
stronger position relative to the buyer.
Suppliers: The cost leadership strategy also allows a firm to better absorb any cost increases forced on it by
powerful suppliers, because the cost leader has greater margins than its competitors. In fact, a cost leader may
be able to force its suppliers to keep prices low for them.
Entrants: The cost leadership strategy also discourages new entrants because the new entrant must be
willing to accept no better than average returns until they gain the experience and core competencies required to
approach the efficiency of the cost leader.
Substitutes: For substitutes to be used, they must not only perform a similar function but also be cheaper
than the cost leader's product. When faced with substitute products, the cost leader can reduce its price.
4. Risks of a cost leadership strategy?
In a cost leadership strategy, the producer seeks to offer products with acceptable features to customers at
the lowest competitive price. One risk of a cost leadership strategy is that the firm's investment in manufacturing
equipment may be made obsolete through technological innovations by competitors. Additionally, a firm with a
cost leadership strategy may focus on cost reduction at the expense of trying to understand customers' needs
and/or competitive concerns. Finally, competitors may be able to imitate a cost leader's competitive advantages
in their own unique strategic actions.
5. Discuss how a differentiation strategy can allow a firm to earn above average returns in spite of
strong competitive forces. Address each of the five competitive forces.
Rivalry: Customers tend to be loyal purchasers of products that are differentiated in ways that are
meaningful to them. As their loyalty to a brand increases, customers' sensitivity to price increases is reduced.
The relationship between brand loyalty and price sensitivity insulates a firm from competitive rivalry.
Buyers: The uniqueness of differentiated goods or services reduces customers' sensitivity to price increases.
Suppliers: Because a firm using the differentiation strategy charges a premium price for its products,
suppliers must provide high-quality components, driving up the firm's costs. However, the high margins the firm
earns in these cases partially insulate it from the influence of suppliers because higher supplier costs can be paid
through these margins. Alternatively, because of buyers' relative insensitivity to price increases, the
differentiated firm might choose to pass the additional cost of supplies on to customers by increasing the price
of its unique product.
Entrants: Customer loyalty and the need to overcome the uniqueness of a differentiated product generate
substantial barriers for potential entrants. Entering an industry under these conditions typically demands
significant investments of resources and patience while seeking customers' loyalty. Substitutes: Firms selling
brand-name goods and services to loyal customers are positioned effectively against product substitutes. In
contrast, companies without brand loyalty face a higher probability of their customers switching either to
products that offer differentiated features that serve the same function (particularly if the substitute has a lower
price) or to products that offer more features and perform more attractive functions.
=> The differentiation strategy is an integrated set of actions taken to produce goods or services (at an
acceptable cost) that customers perceive as being different in ways that are important to them. Product
innovation is critical to successful use of the differentiation strategy. If the firm has a thorough understanding
of what its target customers value, the relative importance they attach to the satisfaction of different needs, and
for what they are willing to pay a premium, the differentiation strategy can be effective in helping it earn above-
average returns.
6. Describe the risks of a differentiation strategy.
https://www.studyblue.com/notes/note/n/chapter-4-business-level-strategy/deck/7118779
https://quizlet.com/272280883/mgmt-4900-midterm-flash-cards/
The risks of a differentiation strategy include the fact that the price differential between the low-cost
producer and the differentiated firm’s product may be too high for the customer. The differentiated products
may exceed the customers’ needs. Additionally, differentiation may cease to provide value for which customers
are willing to pay. This can occur if rivals imitate the firm’s product and offer it at a lower price. A third risk is
that customer learning can narrow the customer’s perception of the value of the firm’s differentiated product. If
customers have positive experience with low-cost products, they may decide the additional cost for the
differentiated product is too high. Finally, counterfeit products are a risk to a differentiation strategy if these
products provide the same differentiated features to customers at significantly reduced prices.
7. How do focused differentiation and focused cost-leadership strategies differ from their non-
focused counterparts?
Focus strategies target specific industry segments or niche rather than the entire market. The market can be
segmented into 1) a particular buyer group, 2) a different part of a product line, or 3) different geographic areas.
The firm using a focus strategy hopes to meet the needs of a particular target market better than firms with a
more broad-based approach. Or, they hope to meet needs of a market niche that has been overlooked or
neglected by broad-based rivals.
8. Describe the additional risks undertaken by firms pursuing a focus strategy.
Focus firms face three additional risks beyond the general risks of industry-wide strategies. First, a
competitor may be able to focus on a more narrowly defined competitive segment and “outfocus” the focuser.
Second, a firm competing on an industry-wide basis may decide the targeted market segment is attractive and
worthy of competitive pursuit. Finally, the needs of the firm’s customer group may become more similar to the
needs of industry-wide customers as a whole, thereby eliminating the advantages of a focus strategy.
9. Describe the advantages of integrating cost leadership and differentiation strategies.
Customers have increasingly high expectations for products, wanting products that are both low-priced and
differentiated. So a number of firms are trying to simultaneously follow both a cost leadership and a
differentiation strategy. This requires the firm to perform the primary and support activities required of both
strategies, which is challenging. Successful integration of strategies allows firms to adapt quickly to
environmental changes, and learn new technologies. The firm gains more skills which makes it more flexible.
Evidence suggests that successful use of integrated strategies is related to above-average returns. A number of
firms such as Target Stores and European-based Zara owe their success to the integrated cost
leadership/differentiation strategy.
10.What are the risks of an integrated cost leadership/differentiation strategy?
Integrated strategies present risks that go beyond those that arise from the pursuit of any single strategy by
itself. Principal among these risks is that a firm becomes “stuck in the middle.” In such a situation a firm fails to
implement either the differentiation or the cost leadership strategy effectively. The firm will not be able to earn
above- average returns, and without favorable conditions, it will earn below-average returns. Recent research
suggests that firms using either cost leadership or differentiation often outperform firms attempting to use a
“hybrid” strategy (i.e., integrated cost leadership/differentiation). This research suggests the risks associated
with the integrated strategy.
Case Scenario 1: International Cow Packers. International Cow Packers (ICP) is a $12 billion meat
processor (slaughter, processing, and packing). Founded in 1943, ICP has grown to become the largest
beef and pork processor in the United States (revenues come 90% from beef and 10% from pork) and
also has a growing export market to Japan. The company follows a focused cost-leadership strategy,
delivering USDA-graded meats primarily to the institutional (schools, prisons, hospitals) and
supermarket channels. ICP's entire value chain is organized to deliver volume product at the industry's
lowest per-unit cost. Its supplier industries, primarily cattle and swine feedlots, have relatively little
power since prices for these raw materials are determined in the commodity markets. While entry
barriers to the industry are high due to high minimum start-up costs, industry rivalry is extremely
intense - primarily due to the fact that three large companies (including ICP) control 80% of the market
for processed meats. The threat of substitutes is high with an increasing trend for consumers to favor
poultry and other non-beef proteins. Buyers are also powerful since supermarkets are relatively
concentrated at a regional level and end-consumers have ample choices. -(Refer to Case Scenario 1)
1. Is ICP's focused low-cost strategy appropriate for its industry? Why?
The best answers will begin by noting that ICP sells a commodity product, as evidenced by the fact that
there are only so many grades of USDA-certified beef and pork. Since the product is an undifferentiated
commodity, customers typically base their purchasing decisions on price alone.
2. What risks is ICP accepting by adopting its focused low-cost strategy?
The best answers will note that since ICP has aligned its entire value-chain with its low-cost strategy, it has
linked its own ups and downs to the ups and downs of the beef and pork industries. Thus, like commodity
prices, we can expect that ICP will do well when general demand for beef and pork is up, and less well when
such demand is down. A more nuanced answer may also point out that ICP’s intense focus on costs may
essentially drive out any opportunities for it to develop differentiation advantages (other than offering the lowest
cost product). If competitors are able to match ICP’s efficiency as well as build other differentiation advantages
(like brand management skills or forward integration into value-added meat products like prepackaged meals),
ICP may find itself at a competitive disadvantage in the long run.
3. What can ICP do to decouple itself from the ups and downs of the pure commodity markets?
What specific actions might ICP undertake?
The best answers will begin by suggesting that ICP must retain its cost advantage while developing
differentiation advantages. At a general industry level, ICP can promote the consumption of beef and pork to
counter trends away from these meats. Specific to ICP, it can begin experimenting with value- added products
like prepackaged meals (frozen dinners, etc.). A related strategy would be the development of organic products
that do not fall within the USDA categories. Selling high-quality beef and pork outside of the USDA categories
would be another strategy as well. The theme across students’ recommendations should be one of developing
products that no longer have commodity-like characteristics.
Case Scenario 2: Walt Disney Company. Walt Disney Company is famed for its creativity, strong
global brand, and uncanny ability to take service and experience businesses to higher levels. In the early
1990s, then-CEO Michael Eisner looked to the fast-food industry as a way to draw additional attention to
the Disney presence outside of its theme parks - its retail chain was highly successful and growing rapidly.
A fast-food restaurant made sense from Eisner's perspective since Disney's theme parks had already
mastered rapid, high-volume food preparation, and, despite somewhat undistinguished food and high
prices (or perhaps because of), all its in-park restaurants were extremely profitable. From this
inspiration, Mickey's Kitchen was launched. The first two locations were opened in California and in a
suburb of Chicago, adjacent to existing Disney stores. Menu items included healthy, child-oriented fare
like Jumbo Dumbo burgers and even a meatless Mickey Burger. Eisner thought that locating each
restaurant next to existing Disney stores was sure to increase foot traffic through both venues. Less than
two years later Disney closed down the California and Chicago stores and shuttered further expansion
plans. Eisner cited overwhelming competition from McDonalds and general oversaturation in the fast-
food industry as the primary reasons for closing down the failing Mickey's Kitchen. -(Refer to Case
Scenario 2)
1. Based on your own knowledge of Disney and the information provided in the scenario, does
Disney appear to create value in its businesses primarily through a cost-leadership or through a
differentiation strategy?
The best answers will begin by noting that Disney, via Mickey Mouse, is probably one of the world’s most
recognized brands. This unique asset complements a differentiation strategy well. Students may further remark
that while Disney may seek efficiencies in all of its operations, ticket prices for the theme parks do not appear to
be a particular bargain, and that Disney never seems to promote its products based on their cost. This is
illustrated by the point that the in-park restaurants charge high prices.
=> Although the Mickey’s Kitchen failed, but Disney still appear to create value in its businesses primarily
through differentiation strategy. Disney’s strategy of differentiated more reflect the value of health in upstream
and downstream link, formed the unique brand value chain operation mode, and realize the years of stable
growth. After nearly 90 years of development, the company has become a Disney film and television
entertainment, with theme park, media network and merchandise of business such as the world's most famous
enterprise of the corpse, brand value, to $29.2 billion.
2. What resources and value-chain activities did Disney try to leverage through the opening of
Mickey's Kitchen?
It appears that Disney was hoping for a differentiation advantage through (1) the image of Mickey Mouse,
(2) its service management expertise, particularly in food service, and (3) locations next to its already successful
chain of retail outlets.
=> Disney is Walter Dean Disney Company's brand. Cartoon characters and fast food industry group is the
Disney in an operation brand portfolio strategy. Because Disney cartoon character is the Disney starting period
of the brand development, continue today. Tourists from cognitive perspective, the movie cartoon characters
such a brand into of fast food industry, can make visitors quickly built "fast food kitchen-cartoon characters-
Disney" brand association contact.
3. Why do you think that Mickey's Kitchen failed?
The best answers will begin with the observation that it is hard to imagine that Mickey’s Kitchen could
create the differentiated Disney experience and margins at fast-food prices. The discussion can then be extended
to note that Disney did deploy a set of resources that were valuable, rare, costly to imitate, and non substitutable,
but it did so in the fast-food industry where consumers make choices based primarily on price. Thus, Disney’s
particular resources generated differentiation advantages, but not the needed cost advantages. It also can be
pointed out that Disney’s theme park restaurants have likely done well because guests of the park are a captive
audience and have few food choice alternatives unless they opt to leave its park or properties.
=> As we all know, Brand combination as a brand strategy important constituent, need to rely on strong
brand as a company, based in different levels, with the scientific method the brand portfolio. So I think the
reason of the Mickey’s Kitchen failed is the unscientific strategy. The unscientific parts are due to
overwhelming competition from McDonalds and Fast food industry is saturated. Competition is the key to the
success or failure of the enterprise. McDonald's corp., the most famous McDonald's brand has more than 32000
home fast-food restaurants, distribution in 121 countries and regions. In the world according to the taste of the
local people to McDonald's meal adjusted. At present, in the global fast food chain McDonald's field is the
champion. In this respect, Mickey’s Kitchen is completely without the advantages. Fast food industry market
saturation is also the important reasons that lead to the collapse. Because the fast-food industry is not Disney’s
main industry, so the resources of relatively that it takes into the resources of relatively is small.
Case Scenario 3: Abrahamson's Jewelers. Through its sole location in an affluent suburb of San
Francisco, Abrahamson's Jewelers has established a strong niche market in the upscale jewelry store
segment. Abrahamson's was founded in 1871 and is currently owned and operated by John Wickersham,
who bought the firm from its namesake founders in 1985. Wickersham joined the firm as a trainee out of
high school, completed his gemology training, and several years later took ownership with the financial
help of his parents. That debt has long been paid off and business has thrived. When he first acquired the
business, Abrahamson's offered a full range of jewelry and gift items from watches to wedding sets to
silverware to clocks. This broad range of products was mirrored by a broad price range-$10,000 Rolex
watches were sold next to $50 Seiko watches. While some jewelry was custom designed and
manufactured, most of the products were "case ready," meaning they were sourced from large jewelry
and silver manufacturers from around the world. Over the last 15 years, Wickersham has narrowed the
company's product offering considerably to focus only on high-end watches like Rolex and Piaget, custom
jewelry, and estate jewelry. Wickersham stresses that this is an appropriate focus for his business since
each of the products lends itself to relationship selling, and price rarely comes into the discussion. Despite
the narrower offering moreover, Abrahamson's floor space has doubled, and clients are intensely loyal to
the good taste, design skills, and personal service level provided by Mr. Wickersham. -(Refer to Case
Scenario 3)
1. What generic business strategy best describes Abrahamson's? Why?
The best answers will observe that all the features of this case point to a focused differentiation strategy.
The company is focused both in terms of product offering and geography. Purchase decisions are based
primarily on a relationship with Mr. Wickersham and unique products, not on price.
2. While Abrahamson's is doing well, Mr. Wickersham would like to grow his business beyond the
present location. He believes that growth may bring greater profitability, as well as employment avenues
for his only child, who will soon be finishing high school. What recommendations do you have for Mr.
Wickersham regarding his growth choices?
The objective here is to get students to see the limits to growth presented by Abrahamson’s current strategy
and key resources. This scenario also provides a nice opportunity to link a company’s strategy and resource base
with a key individual—in this instance, Mr. Wickersham. The best answers will start by walking through a
particular expansion plan and then noting how the company’s resources do and do not support that plan. For
instance, one obvious avenue is to open additional locations. Such an avenue would likely leverage
Wickersham’s contacts and expertise in sourcing raw materials, as well as providing a greater market to exploit
his representation and contracts with watch firms like Rolex and Piaget. A second avenue would be to leverage
Wickersham’s design skills to go into the wholesale jewelry business. The risk underlying both of these growth
avenues is that it may spread Mr. Wickersham too thin: as the scenario clearly suggests, his personal knowledge
and relationships (and time) are central to Abrahamson’s current success.
3. Would you recommend that Mr. Wickersham embark on an Internet sales strategy for his
company?
The best answers will note that some aspect of the Internet may be valuable for Abrahamson’s, but that his
current resource base does not lend itself well to an Internet sales vehicle. Customers typically expect that
products sourced and sold online will be cheaper than through traditional retail channels, even for high-end
items like watches (for instance, have students do a Web search for Rolex watches). Abrahamson’s is not poised
to, nor does it seem inclined to, compete on price. In terms of customer relations, however, Abrahamson’s could
use some form of Internet presence to show its customers a broader variety of products in its already narrow
line. They could also perhaps see prior design work to help them better imagine what a custom-designed piece
might look like. Particularly for the estate sales, Abrahamson’s could link its inventory to larger, reputable
online estate sale houses—thus giving its customers the benefit of local relationships with the power of the
Internet’s worldwide markets. Finally, use of the Internet for maintaining contact with existing customers would
enhance its relationship with and knowledge of them.

Quiz 5: Competitive Rivalry and Competitive Dynamics


https://wenku.baidu.com/view/fc888a8602d276a200292ec7.html
1. Define competitors, competitive rivalry, competitive behavior, and competitive dynamics.
Competitors are firms competing in the same market, offering similar products, and targeting similar
customers.
Competitive rivalry is the ongoing set of competitive actions and competitive responses occurring between
competitors as they compete against each other for an advantageous market position.
For the individual firm, the set of competitive actions and responses it takes while engaged in competitive
rivalry is called competitive behavior.
Competitive dynamics is the set of actions and responses taken by all firms that are competitors within a
particular market
2. What is market commonality? What is resource similarity? How are these concepts combined to
identify the level of competition between two firms?
Market commonality is concerned with the number of markets with which the firm and a competitor are
jointly involved and the degree of importance of the individual markets to each. Each industry is composed of
several markets which are characterized by different customers, product segments, or geographic areas, among
other differentiators. In general, competitors agree about the different characteristics of the individual markets
that make up an industry. Most industries’ markets are somewhat related in terms of technologies used or core
competencies needed to develop a competitive advantage. Firms may compete against one another in one
market or in many, a situation termed multimarket competition.
Resource similarity is the extent to which the firm’s tangible and intangible resources are comparable to a
competitor’s in terms of both type and amount. Firms with resource similarity are likely to have similar
strengths and weaknesses and to use similar strategies.
The combination of high or low market commonality and high or low resource similarity identifies whether
firms are competitors. Firms having both high market commonality and high resource similarity are direct and
mutually acknowledged competitors. If firms share few markets and have little similarity in resources they are
not direct and mutually acknowledged competitors.
3. Define awareness, motivation and ability in reference to competitive behavior.
Awareness, motivation and ability are the drivers of competitive behavior. They influence the firm’s
actions toward and responses to competitors. Awareness is the extent to which competitors recognize the degree
of their mutual interdependence that results from market commonality and resource similarity. Awareness
affects the extent to which the firm understands the consequences of its competitive actions and responses.
Awareness is greatest when firms have highly similar resources. Motivation concerns the firm’s incentive
to take action against a competitor or to respond to a competitor’s attack. If the firm doesn’t believe that
attacking its competitors will improve its position, it will not act. If the firm does not believe a competitor’s
action will result in losses for it, it will not have motivation to respond. High market commonality gives firms
more motivation to attack and to respond to competitors’ actions than when market commonality is low. Ability
relates to each firm’s resources and the flexibility these resources provide. When a firm faces a competitor with
similar resources, careful study of a possible attack is essential because a competitor with similar resources is
likely to respond to competitive attack. When the resources between two competitors are very dissimilar, the
weaker firm will delay in responding to an attack by the stronger firm.
4. Define competitive actions and responses and explain the two types of competitive actions and
responses.
This refers to strategic and tactical actions. Strategic actions take more time to implement, require many
specific resources, and are difficult to reverse. Tactical actions tend to be quicker to implement, require fewer
resources, and can be reversed more easily. Strategic actions tend to receive strategic responses. Tactical actions
tend to receive tactical responses, because they are easy to put into place. Strategic actions elicit fewer total
competitive responses than do tactical actions. Responses to strategic actions will be slower than will responses
to tactical actions because competitors need time to observe whether the strategic action will be successful. But,
if a competitor’s action threatens a large number of a firm’s customers, the firm will react strongly regardless of
whether the competitor’s action is strategic or tactical.
5. What are the advantages and disadvantages of being a first mover, second mover, and late
mover?
First movers can gain market share, customer loyalty, and high revenues by being the first in the market.
But, first movers also take more risk because it is difficult to judge the returns the firm will earn from product
innovations. Moreover, if the first mover is successful, other firms will enter its arena. First movers tend to have
a significant amount of organizational slack to fund research and development. Second movers imitate the first
movers, after they have studied the first mover’s successes and mistakes. Consequently second movers can
develop more efficient processes and technologies than first movers, which results in lower costs. Late movers
react to the first and second movers’ actions after a long delay. A late mover may be able to earn average returns
if it has learned how to create at least as much value for customers as the value created by the first and second
movers. In general, late movers are relatively ineffective.
6. What factors contribute to the likelihood of a response to a competitive action?
In general, a firm is more likely to respond to a competitive action if: (1) the action leads to better use of
the competitor’s capabilities to gain or produce stronger competitive advantage or to improve its market
position, (2) the action damages the firm’s ability to use its capabilities to create or maintain an advantage, or
(3) the firm’s market position becomes less defensible. In addition, a firm is more likely to respond to a
competitor’s tactical action, rather than to a competitor’s strategic action. Strategic actions involve a significant
commitment of resources and are difficult to implement and reverse, as well as requiring time to put into place.
In contrast, tactical actions can be implemented quickly and are quickly reversed, and are relatively less costly
than strategic actions. A firm is also more likely to respond to a competitor’s action when the competitor is the
market leader - a firm that has the reputation for above-average returns. Successful actions by competitors are
likely to be quickly imitated, even if not initiated by a market leader. Actions by price predators are usually not
responded to, nor are actions by firms with reputations for risky, complex, and unpredictable behavior. Finally,
competitors with high market dependence are likely to respond strongly to attacks threatening their market
position.
7. Define slow-cycle, fast-cycle and standard cycle markets.
In slow-cycle markets, the firm’s competitive advantage is shielded from imitation for long periods of time
and imitation is costly. Competitive advantages are sustainable in slow-cycle markets. Successful firms in slow-
cycle markets have difficult-to-understand and costly-to-imitate advantages resulting from unique historical
conditions, causal ambiguity and/or social complexity. These conditions can include copyrights, patents, and
ownership of an information resource. Firms in slow-cycle markets focus on protecting their competitive
advantages and exploiting them as long as possible. In fast-cycle markets, imitation happens quickly.
Competitive advantages are not sustainable. Reverse engineering and quick technology diffusion facilitate rapid
imitation. In fast-cycle markets, innovation is critical and firms avoid “loyalty” to any product. Firms must
focus on rapidly and continuously developing new competitive advantages, because prices fall quickly and firms
need to profit rapidly from innovations, and move on to the next product. Fast-cycle markets are volatile and the
pace of innovation is frenzied. In standard-cycle markets, the firm’s competitive advantages are moderately
shielded from imitation and imitation is moderately costly. Competitive advantages are partially sustainable if
the firm can continuously upgrade the quality of its capabilities making its competitive advantage dynamic.
Typically, these markets have large firms seeking high market share, striving for customer brand loyalty, and
controlling their operations to give customers consistent experiences. Economies of scale are necessary for
survival. Competition for market share is intense and is often based on incremental innovation in a product
rather than radical innovation.
Case Scenario 1: Romulac, Inc. Romulac Inc. (RI), a subsidiary of a large successful manufacturing
conglomerate, supplies a key component in the assembly of residential cooling systems (air conditioning
units, etc.). There has been tremendous consolidation in RI's industry, to the point where only five
suppliers of this particular component account for nearly 90% of U.S. industry sales. Paralleling this
trend, its customers - comprised of makers of branded residential air conditioning units like Carrier and
Trane - have seen similar levels of consolidation in their own industry. Half of these firms produce all
their components in-house, while the balance purchases them from specialized component manufacturers
like RI. RI's business is extremely capital intensive, and their 40% share of the market allows them to
also be the most profitable domestic player. Strong competitors exist in Europe and Asia. Although like
RI, these foreign players' strongholds are their home regions, with negligible presence outside of the
region. Some of the larger Asian manufacturers have signaled an interest in more aggressively pursuing
the lucrative U.S. market. RI is presently considering a $400 million dollar investment in a new plant,
which will create a component that is much quieter, more efficient, and is likely to satisfy future
regulatory standards. While the core technology for the new component is very old, RI's engineering and
design skills have allowed them to retain their low cost-advantage, even though the component will
represent a significant improvement over products currently provided by its competition. -(Refer to Case
Scenario 1)
1. Develop an argument as to why RI should try to be a first-mover with this new technology.
The best answers will begin by suggesting that RI move quickly to retain its dominant market share,
particularly since the technology itself is not new, and competitors may easily develop their own efficient
designs. A more subtle argument is that RI has an opportunity to set a new industry standard,and as the leader,
may likely gain even greater market share.
2. Develop an argument as to why RI should hold back and be a second mover with the new
technology.
The best answers will observe that RI cannot predict with certainty that its new technology will be cheaper,
or as cheap to produce. Thus, a primary risk is that RI invests in the plant, the industry moves to the new
technology, but the technology is actually more costly to produce - in this way, RI may cannibalize its existing
low-cost position with a higher-cost one. By waiting, RI can learn from its competitors’ mistakes. A secondary
risk is that competitors will learn from RI’s initial mistakes, and be able to offer the new technology for
considerably lower cost. The nightmare scenario here is that the industry moves to the new technology, RI has a
higher cost position, and overseas competitors steal domestic market using the new technology, which they have
learned to manufacture at a lower cost. Thus, RI could avoid this latter risk by again waiting out the
competition.
3. As one of RI's direct competitors, how would you try to predict what it will do with regard to the
new technology?
The best responses can begin by pointing to three main characteristics that are likely to heavily influence
RI’s choices. The ways these factors bode in favor and against the move should be discussed. First, RI is both
large and a dominant player in this market. Second, RI is very profitable and such profitability is a direct
consequence of its large market share. Finally, RI is the subsidiary of a large successful conglomerate - students
would want to point out that the new technology will require a large corporate commitment ($400 million). A
related issue is whether the corporate parent considers itself to be primarily a first or second mover in its
competitive interactions.
Case Scenario 2: Plasco. Plasco is a $3 billion U.S.-based manufacturer of flexible plastic products
like trash cans, reheatable and freezable food containers, and a broad range of other plastic storage
containers designed for home and office use. Historically, Plasco has been the category killer for most of
its products and has devoted tremendous resources to new product development on an ongoing basis -
this research intensity has allowed the company to release, on average, a new product every day over the
past five years. Despite its past strength and high brand awareness, Plasco's profitability has been eroded
by dramatic increases in the cost of plastic resin, the primary input into its plastic products. Moreover,
the retail channel has experienced rapid consolidation resulting in a shift in the balance of power from
branded manufacturers like Plasco, to strong retailers like Wal-Mart, who in turn have been unwilling to
help Plasco absorb the higher resin costs. Enhancing Wal-Mart's power is the fact that it can always turn
to alternative high-volume sources of consumer plastic products like Sterlite. Further hampering Plasco's
recovery is the emergence of feisty little foreign competitors like Zig Industries, a $250 million Israeli
firm that has begun to take part of Plasco's market share in plastic toolboxes. Ironically, Plasco was the
first company to offer plastic toolboxes some 20 years ago. This innovation changed the market
dramatically and Plasco's first mover strategy rewarded it with a rapidly growing new segment and a
dominant market position. Today, Plasco's toolboxes are viewed as rather boring, while Zig's products
are ingeniously designed to catch the customer's eye in the aisle (better merchandising the product) and
capture their interest (and pocketbook) with many new and novel features. Zig is also able to provide this
new line of toolboxes at between 10% to 15% less than Plasco. -(Refer to Case Scenario 2)
1. Is Walmart a competitor or a customer of Plasco?
The best answers will start by summarizing that Wal-Mart is both a customer and a competitor. It is a
customer in the sense that it is a primary outlet for Plasco’s products. Wal-Mart is a competitor from the
standpoint that Walmart has control over Plasco’s profitability and, in a sense, is competing for a portion of the
profit pool in Plasco’s industry as well.
2. Is the toolbox business a slow-, standard-, or fast-cycle business?
The best answers will suggest that the cycle characteristics of the toolbox market appear to have shifted
over time. Before Plasco entered the business, metal toolboxes were the norm and this was likely to be
characterized as standard to slow in terms of its cycle speed. The metal toolbox market was probably an
oligopolistic one, dominated by a few profitable players. With the entrance of Plasco and its plastic toolboxes,
the cycle speed among metal toolbox manufacturers increased, where they no longer dominated the industry.
Plasco’s innovative product plus its unique capacity (at the time) to produce a durable plastic toolbox probably
turned its segment into a slow-cycle market (with Plasco enjoying a near monopoly position), while the metal
toolbox took on standard- to fast-cycle market characteristics. With the new entrance of Zig into plastic
toolboxes, this segment is now likely to be characterized as standard to fast cycle - plastic technologies aren’t
proprietary and designs are readily copied by competitors.
3. How can a small player like Zig be such a successful competitor against a large, established firm
like Plasco?
The value of this question is that it forces students to consider how changes in both a focal industry and its
upstream industry may affect competition. The best answers can begin by noting that the market for plastic
toolboxes is probably pretty large (tools, cosmetics, fishing gear, toys, etc), especially on a global basis. Couple
this observation with the fact that there are a number of mega-retailers who would find this to be a necessary
product to stock on their shelves. Thus, this market segment is nearly an industry in and of itself and a small
focused player could gain economies of scale in manufacturing as well as distribution and marketing to the
staple, volume retailers like Wal-Mart, Home Depot, and Carrefour. Finally, Plasco was apparently treating the
plastic toolbox market as a stable one, leaving it less likely to invest much of any additional resources into
further innovation. This creates a window of opportunity for a nimble, aggressive, focused and talented new
entrant like Zig.
Quiz 6: Corporate-Level Strategy
https://www.studyblue.com/notes/note/n/chapter-6-corporate-level-
strategy/deck/7118898#:~:text=The%20five%20categories%20of%20businesses%20determined
%20by%20level%20of%20diversification,per%20cent%20of%20revenue%20from
https://quizlet.com/84790104/essay-questions-for-chapter-6-flash-cards/
1. Differentiate between corporate-level and business-level strategies and give examples of each.
A business-level strategy determines how a firm will compete in a single industry or product market. When
a firm diversifies beyond a single industry it uses a corporate- level strategy. A diversified company has two
levels of strategy: business-level and corporate-level. Each business unit has a business level strategy. The
corporate strategy is concerned with: 1) what businesses the firm should be in and 2) how the corporate office
should manage the group of businesses. The top management of diversified companies views the firm's
businesses as a portfolio of core competencies that will generate above-average returns by creating value. An
example of a business-level strategy would be whether the firm targets the mass market and competes on price,
or whether it competes on the basis of uniqueness. An example of a corporate-level strategy would be whether
the firm should sell off a poorly performing subsidiary.
2. What are the five categories of businesses based on level of diversification?
The five categories of businesses determined by level of diversification are as follows: (1) Single business
(more than 95 percent of revenues from a single business), (2) Dominant business (between 70 percent and 95
percent of revenue from a single business), (3) Related constrained (a diversified organization earning less than
70 percent of revenue from the dominant business, and all the component businesses share product,
technological, and distribution linkages), (4) Related linked (a diversified organization earning less than 70
percent of revenues from the dominant business with only limited links among the component businesses), and
(5) Unrelated (diversified organizations earning less than 70 percent of revenues from the dominant business
with no common links among the businesses).
3. Describe the primary reasons a firm pursues increased diversification.
Firms typically diversify to increase the firm's value by improving its overall performance. Value-creating
diversification occurs through related or unrelated diversification when the strategy allows the company's
business units to increase revenues or reduce costs while implementing business level strategies. Alternatively, a
firm may diversify to gain market power over competitors. Value-neutral diversification may occur in response
to governmental policies, firm performance problems, or uncertainties about future cash flows. Finally,
managers may have selfish motives to diversify, such as increased compensation or personal reduced
employment risk. These selfish motivations may actually erode the firm's competitiveness, and can be value-
reducing diversifications.
4. Describe how diversified firms can use activity sharing and transfer of core competencies to
create value.
In related diversification, a firm seeks to exploit economies of scope between its business units. Economies
of scope are cost savings created by transferring some of its capabilities and competencies developed in another
business to a new business. Firms create value through economies of scope two ways: the sharing of activities
(operational relatedness) and the transferring of core competencies (corporate relatedness). Both primary and
support activities may be shared, including marketing and production. This activity sharing can result in cost
reductions and improve financial returns. The sharing of core competencies allows the firm to create value two
ways: 1) it eliminates the need for the second unit to allocate resources to develop the competence, and 2)
transferring intangible resources internally makes it hard for competitors to understand and to imitate the
resource.
5. What are the two ways that an unrelated diversification strategy can create value?
Unrelated diversification can create value through two types of financial economies (cost savings). 1)
Unrelated diversified firms can more efficiently allocate capital among the component businesses than can the
external financial market. This is possible because the corporate-level management has more complete
information about the performance of the component businesses and it can also discipline under- performing
management teams. 2) Unrelated diversified firms can also create value by purchasing other businesses at low
prices, restructuring them, and reselling them at a higher price. This practice is most successful with mature,
low-technology businesses, rather than high-technology or service businesses, which are more dependent on
employees who may leave.
6. What is the effect of a firm's low performance on the pursuit of diversification?
High corporate performance eliminates the need for diversification. Some research shows that low returns
are related to greater levels of diversification. Firms plagued by poor performance often diversify in an effort to
become more profitable. But, continued poor performance following diversification may slow the pace of
diversification and may lead to divestitures and a focus on the core business. In addition, firms that are more
broadly diversified compared to their competitors may have lower overall performance. Figure 6.3 shows that
the related constrained diversification strategy is the highest performing strategy. So poor performing firms that
intend to diversify should look at purchasing businesses that would be suitable for this strategy rather than
moving into unrelated diversification or retaining a dominant business strategy.
7. What are the managerial motives to diversify?
A top-level manager may be motivated to pursue diversification because diversification leads to greater job
security for executives. In general, greater amounts of diversification reduce managerial risk because if a
particular business fails, the top executive remains employed by the corporation. In addition, diversification
increases firm size, and firm size has a direct effect on executive compensation. Moreover, managing a highly
diversified firm is more difficult; thus, managerial compensation is generally higher in such a firm.
Consequently, executives may have selfish motives to diversify the company in ways which may actually
reduce corporate competitiveness.
Case Scenario 1: Syco. Syco is a diversified company that has six primary lines of business. Fifty
percent of its revenues and 18 percent of its profits come from retailing. Most of its retail outlets are
discount department stores that serve as anchor tenants for large suburban shopping malls. The
remaining businesses are broken out as follows: Insurance accounts for 30 percent of revenues and 50
percent of profits; consumer credit card operations are 6 percent of sales and 17 percent of profits; 5
percent of revenues and 6 percent of profits come from its stock brokerage business; commercial and
residential real estate operations generate 4 percent of sales and 8 percent of profits; finally, 5 percent of
revenues and 1 percent of profits come from its online portal business. The company's management states
that all these businesses are essential to its competitive future. -(Refer to Case Scenario 1)
1. Why might there be so much variability among the proportion of sales versus profitability
contributed by each of the businesses? Does this mean that Syco is more successful in its insurance
business than in its retail business?
The best answers to this question will start out by noting that industries vary in their profit structures. That
is the margins in retailing are typically very low, while those in insurance are relatively much higher. Beyond
this, the statement tells us nothing about how well Syco is actually doing in each business, since you would need
to compare business-level performance against that of competitors on a business-by-business basis. [Note to the
instructor: The above scenario is based loosely on Sears in the mid-1980s. At that time its lines of business were
Sears Department Store, Allstate Insurance, Discover Card, Dean Witter Stock Brokerage, Coldwell Banker
Real Estate Brokers, and Prodigy Online. Sears eventually divested all but its department stores and, at one
time, was near bankruptcy under the weight of its diverse operations and failing retail business (under-
maintained mall properties became a core rigidity).]
2. Part 1: (Refer to Case Scenario 1) Develop a logical argument that would lead you to describe
Syco's diversification type as related linked and another logical argument that Syco's diversification type
is related constrained. For both the related linked and for the related constrained arguments, what
product, technological, or distribution activities might link these businesses together? Part 2: Would you
describe either of the logical arguments you developed in response to Part 1 as a good corporate strategy?
Part 1: The purpose of this exercise is to give students an opportunity to view the diversified firm from the
perspective of the top management team. The best answers will start with related linked strategies and note that
superficial linkages can be provided by global brandings (i.e., all businesses under one name), one-stop
shopping (i.e., all businesses under one roof), and shared accounting systems (i.e., centralized accounting, cash
allocation, and planning). The best answers to the related constrained questions will offer more complex
linkages beyond those noted above like shared customer lists, active cross-selling, and rotation of key personnel.
Because the businesses are so different, however, the students should begin to see that more complex linkages
may be difficult to achieve.
Part 2: Based on the challenges of developing answers for Part 1, this question forces students to stand back
and criticize the strategies that they created. Since there are many possible approaches to Part 1, it should
become clear to students that each of these competing arguments has significant drawbacks. Ultimately,
students should reach the point where they may agree that offering a bundle of services is desirable, although
there is no reason that Syco has to own all of these offerings. This discussion also provides a nice way to
foreshadow the role and importance of strategic alliances which are covered in Chapter 9.
Case Scenario 2: Jewell Company. Jewell Company (JC) is a $2 billion diversified manufacturer and
marketer of simple household items, cookware, and hardware. In the early 1950s, JC's business consisted
solely of manufactured curtain rods that were sold through hardware stores and retailers like Sears.
Since the 1960s however, the company has diversified extensively through acquisition into such businesses
as paintbrushes, writing pens, pots and pans, and hairbrushes. Over 90 percent of its growth can be
attributed to these many small acquisitions, whose performance it improved tremendously through
aggressive restructuring and its corporate emphasis on cost-cutting and cost controls. While JC's sixteen
different lines of business may appear quite different, they all share the common characteristics of being
staple manufactured items and sold primarily through volume retail channels like Wal-Mart, Target, and
Kmart. Because JC operates each line of business autonomously (separate manufacturing, R&D, and
selling responsibilities for each line), it is perhaps best described as pursuing a related linked
diversification strategy. The common linkages are both internal (accounting systems, product
merchandising skills, and acquisition competency) and external (distribution channel of volume
retailers). JC is presently contemplating the acquisition of Plastico, a $3 billion U.S.-based manufacturer
of flexible plastic products like trash cans, reheatable and freezable food containers, and a broad range of
other plastic storage containers designed for home and office use. While Plastico has been highly
innovative (over 80% of its growth has come from internal new product development), it has had
difficulty controlling costs and is losing ground against powerful customers like Wal-Mart. JC believes
that the market power it wields with retailers like Wal-Mart will help it turn Plastico's prospects around.
-(Refer to Case Scenario 2) https://wenku.baidu.com/view/2377e989f90f76c660371a19.html
1. How might JC's related diversification strategy result in economies of scope and market power?
For the question of efficiencies, the best answers will observe that:
(1) JC is able to leverage its accounting systems, product merchandising skills, acquisition competency, and
distribution channel of volume retailers across all its businesses (economies of scope).
(2) These scope economies, in turn, provide JC the opportunity to build scale economies in these resources
and value chain activities.
(3) The market power dimension comes into play in terms of the scope economies it realizes within the
channel-for instance, JC's multi-product portfolio is likely to give it more power with regard to Wal-Mart than
would be possessed by a single product firm like Plastico.
=> Since the 1960s however, the company has diversified extensively through acquisition into such
businesses as paintbrushes, writing pens, pots and pans, and hairbrushes. Over 90 percent of its growth can be
attributed to these many small acquisitions, whose performance it improved tremendously through aggressive
restructuring and its corporate emphasis on cost-cutting and cost controls. It is perhaps best described as
pursuing a related linked diversification strategy.
2. Why would the acquisition of Plastico be good for JC?
The best answers will note that (1) JC has likely created some market power with respect to the large
retailers so another staple consumer product makes sense. (2) JC can leverage its existing market presence,
selling contacts, distribution system, and merchandising skills for plastic consumer products as well. (3) And
since Plastico is having trouble controlling its costs, JC can bring to it more sophisticated financial management
skills and accounting systems.
=> Plastico, a $3 billion U.S.-based manufacturer of flexible plastic products like trash cans, reheatable and
freezable food containers, and a broad range of other plastic storage containers designed for home and office
use. The acquisition of Plastico can help JC enlarge industrial diversification. JC was already able to different
business through the retail channel together, expand their marketing advantage. From this, JC believes that the
market power it wields with retailers like Wal-Mart will help it turn Plastico’s prospects around.
3. What difficulties might you expect JC to encounter related to its acquisition of Plastico?
The best answers will begin by quickly noting that (1) Plastico is much bigger than JC, which itself is
likely to create problems (a nice lead-in to chapter 7, Acquisition and Restructuring). (2) And while JC has
grown through acquisition, it has primarily made small ones, which suggests that the firm may be ill-prepared
for the enormous challenge of taking control of such a large firm. (3) And even though Plastico appears to be in
trouble controlling costs, a more subtle observation is that JC's cost control emphasis may be too extreme for
Plastico's historic competency in innovation and, ultimately, undermine this valuable resource.
=> The main difficulty is other takeover of the competition. Because of other investors to join, the
acquisition of cost will increase. Meanwhile, the JC sales in this aspect of the advantage may be affected by the
impact of other companies. Next is the acquisition of the timing of. The company is the first step of options to
buy the right time. In fact, for the company to make acquisitions in goal, to their own have a clear, reasonable
assessment of the target company has a clear orientation. At the same time, also analysis the macroeconomic
environment, legal environment and social environment, etc. On the other hand, the purchase risk analysis is
also a problem. Company purchase is high risk management, the purchase risk very complicated and widely, the
company should be treated with caution, try to avoid risk, the risk in the acquisition and eliminate every aspect
of, in order to purchase success. In a word, in a takeover, to buy a company mainly face the risk: the market risk
and operational risk, the risk and financing risk, lose legal risk, and integrate the risk, etc.
Quiz 7: Merger and Acquisition Strategies
https://quizlet.com/61443725/busa-ch-7-flash-cards/
https://quizlet.com/85342340/strategic-management-ch-7-essay-flash-cards/
1. How have changing conditions in the external environment influenced the type of M & A activity
firms pursue?
During the recent financial crisis, tightening credit markets made it more difficult for firms to complete
mega deals (those costing $10 billion or more). As a result, many acquirers focused on smaller targets with a
niche focus that complemented their existing businesses. In addition, the relatively weak U.S. dollar increased
the interest of firms from other nations to acquire U.S. companies.
2. How difficult is it for merger and acquisition strategies to create value and which firms benefit
the most from M & A activity?
Evidence suggests that using merger and acquisition strategies to create value is challenging. This is
particularly true for acquiring firms in that some research results indicate that shareholders of acquired firms
often earn above-average returns from acquisitions while shareholders of acquiring firms typically earn returns
that are close to zero. In addition, in approximately two-thirds of all acquisitions, the acquiring firm's stock price
falls immediately after the intended transaction is announced. This negative response reflects investor's
skepticism about the likelihood that the acquirer will be able to achieve the synergies required to justify the
premium.
3. Identify and explain the seven reasons firms engage in an acquisition strategy.
(1) Increased market power. Market power allows a firm to sell its goods or
services above competitive levels or when the costs of its primary or support activities are below those of
its competitors. Market power is derived from the size of the firm and the firm's resources and capabilities to
compete in the marketplace. Firms use horizontal, vertical, and related acquisitions to increase their size and
market power.
(2) Overcoming entry barriers. Firms can gain immediate access to a market by purchasing a firm with an
established product that has consumer loyalty. Acquiring firms can also overcome economies of scale entry
barriers through buying a firm that has already successfully achieved economies of scale. In addition,
acquisitions can often overcome barriers to entry into international markets.
(3) Reducing the cost of new product development and increasing speed to market. Developing new
products and ventures internally can be very costly and time consuming without any guarantee of success.
Acquiring firms with products new to the acquiring firm avoids the risk and cost of internal innovation. In
addition, acquisitions provide more predictable returns on investments than internal new product development.
Acquisitions are a much quicker path than internal development to enter a new market, and they are a means of
gaining new capabilities for the acquiring firm.
(4) Lower risk compared to developing new products internally. Acquisitions are a means to avoid internal
ventures (and R&D investments), which many managers perceive to be highly risky. However, substituting
acquisitions for innovation may leave the acquiring firm without the skills to innovate internally.
(5) Increased diversification. Firms can diversify their portfolio of business through acquiring other firms.
It is easier and quicker to buy firms with different product lines than to develop new product lines
independently.
(6) Reshaping the firm's competitive scope. Firms can move more easily into new markets as a way to
decrease their dependence on a market or product line that has high levels of competition.
(7) Learning and developing new capabilities. By gaining access to new knowledge, acquisitions can help
companies gain capabilities and technologies they do not possess. Acquisitions can reduce inertia and help a
firm remain agile.
4. Describe the seven problems in achieving a successful acquisition.
(1) Integration difficulties. It may be difficult to effectively integrate the acquiring and acquired firms due
to differences in corporate culture, financial and control systems, management styles,
and status of executives in the combined firms. Turnover of key personnel from the acquired firm is
particularly negative.
(2) Inadequate evaluation of target. Due diligence assesses where, when, and how management can drive
real performance gains through an acquisition. Acquirers that fail to perform effective due diligence are likely to
pay too much for the target firm.
(3) Large or extraordinary debt. Acquiring firms frequently incur high debt to finance the acquisition. High
debt may prevent the investment in activities such as research and development, training of employees, and
marketing that are required for long-term success. High debt also increases the risk of bankruptcy and can lead
to downgrading of the firm's credit rating.
(4) Inability to achieve synergy. Private synergy occurs when the acquiring and target firms' assets are
complementary in unique ways, making this synergy difficult for rivals to understand and imitate. Private
synergy is difficult to create. Transaction costs are incurred when firms seek private synergy through
acquisitions. Direct transaction costs include legal fees and investment banker charges. Indirect transaction costs
include managerial time to evaluate target firms, time to complete negotiations, and the loss of key managers
and employees following an acquisition. Firms often underestimate the indirect transaction costs of an
acquisition.
(5) Too much diversification. A high level of diversification can have a negative effect on
the firm's long-term performance. For example, the scope created by diversification often causes managers
to rely on financial controls rather than strategic controls because the managers cannot completely understand
the business units' objectives and strategies. The focus on financial controls creates a short-term outlook among
managers and they forego long-term investments. Additionally, acquisitions can become a substitute for
innovation, which can be negative in the long run.
(6) Managers overly focused on acquisitions. Firms that become heavily involved in acquisition activity
often create an internal environment in which managers devote increasing amounts of their time and energy to
analyzing and completing additional acquisitions. This detracts from other important activities, such as
identifying and taking advantage of other opportunities and interacting with importance external stakeholders.
Moreover, during an acquisition, the managers of the target firm are hesitant to make decisions with long-term
consequences until the negotiations are completed.
(7) Growing too large. Acquisitions may lead to a combined firm that is too large, requiring extensive use
of bureaucratic controls. This leads to rigidity and lack of innovation, and can negatively affect performance.
Very large size may exceed the efficiencies gained from economies of scale and the benefits of the additional
market power that comes with size.
5. Describe how an acquisition program can result in managerial time and energy absorption.
Typically, a substantial amount of managerial time and energy is required for acquisition strategies if they
are to contribute to a firm's strategic competitiveness. Activities with which managers become involved include
those of searching for viable acquisition candidates, completing effective due diligence processes, preparing for
negotiations and managing the integration process after the acquisition is completed. Company experience
shows that participating in and overseeing the acquisition activities can divert managerial attention from other
matters that are linked with long- term competitive success (e.g., identifying and acting on other opportunities,
interacting effectively with external stakeholders).
6. What are the attributes of a successful acquisition program?
(1) The acquired firm has assets or resources that are complementary to the acquiring firm's core business.
(2) The acquisition is friendly.
(3) The acquiring firm conducts effective due diligence to select target firms and evaluates the target firm's
health (financial, cultural, and human resources).
(4) The acquiring firm has financial slack.
(5) The merged firm maintains low to moderate debt.
(6) The acquiring firm has sustained and consistent emphasis on R&D and innovation.
(7) The acquiring firm manages change well and is flexible and adaptable.
7. What is restructuring and what are its common forms?
Restructuring refers to changes in a firm's portfolio of businesses and/or financial structure. There are three
general forms of restructuring:
(1) Downsizing involves reducing the number of employees, which may include decreasing the number of
operating units.
(2) Downscoping entails divesting, spinning-off, or eliminating businesses that are not related to the core
business. It allows the firm to focus on its core business.
(3) A leveraged buyout occurs when a party (managers, employees, or an external party) buys all the assets
of a (publicly traded) business, takes it private, and finances the buyout with debt. Once the transaction is
complete, the company's stock is no longer publicly traded.
8. What are the differences between downscoping and downsizing and why are each used?
Downsizing is a reduction in the number of employees. It may or may not change the composition of
businesses in the company's portfolio. In contrast, the goal of downscoping is to reduce the firm's level of
diversification. Downsizing is often used when the acquiring form paid too high a premium to acquire the target
firm or where the acquisition created a situation in which the newly formed form had duplicate organizational
functions such as sales or manufacturing. Downscoping is
accomplished by divesting unrelated businesses. Downscoping is used to make the firm less diversified and
allow its top-level managers to focus on a few core businesses. A firm that downscopes often also downsizes at
the same time.
9. What is an LBO and what have been the results of such activities?
Leveraged buyouts (LBOs) are a restructuring strategy. Through a leveraged buyout, a (publicly traded)
firm is purchased so that it can be taken private. In this manner,
the company's stock is no longer publicly traded. LBOs usually are financed largely through debt, and the
new owners usually sell off a number of assets. There are three types of LBOs: management buyouts (MBOs),
employee buyouts (EBOs), and whole-firm buyouts. Because they provide managerial incentives, MBOs have
been the most successful of the three leveraged buyout types. MBOs tend to result in downscoping, an increased
strategic focus, and improved performance.
10.What are the results of the three forms of restructuring?
Downsizing usually does not lead to higher firm performance. The stock markets tend to evaluate
downsizing negatively, as investors assume downsizing is a result of problems within the firm. In addition, the
laid-off employees represent a significant loss of knowledge to the firm, making it less competitive. The main
positive outcome of downsizing is accidental, since many laid-off employees become entrepreneurs, starting up
new businesses. In contrast, downscoping generally improves firm performance through reducing debt costs and
concentrating on the firm's core businesses. LBOs have mixed outcomes. The resulting large debt increases the
financial risk and may end in bankruptcy. The managers of the bought-out firm often have a short-term and risk-
averse focus because the acquiring firm intends to sell it within 5 to 8 years. This prevents investment in R&D
and other actions that would improve the firm's core competence. But, if the firms have an entrepreneurial
mindset, buyouts can lead to greater innovation if the debt load is not too large.
Case Scenario 1: Syco Inc. (SI). Syco, Inc. (SI) was founded the late 1800s and grew through
acquisition from being primarily a large discount retailer into a highly diversified firm. Beyond retailing
(still SI's dominant business), by the middle of the 1990s its lines of business included significant market
positions in insurance, consumer credit cards, stock brokerage, commercial and residential real estate
brokerage, and an online Internet portal. Each of the non-retail businesses was average in its relative
industry performance. Consistent with the decentralized structure at SI and arms-length corporate
oversight, each of these businesses was also rapidly developing their own unique brands and customer
following. However, within a short period of time it became apparent that the retail business was failing.
SI's vast mall-based department store holdings were suffering from deferred maintenance and
merchandising that did not appear to be popular with its once large consumer base. At the same time,
highly efficient and focused low-cost competitors like Wal-Mart were beginning to take significant
market share from SI. On the verge of bankruptcy by early 2000, SI's management chose to sell off its
insurance, real estate and stock brokerage units; it also spun off its credit card and portal businesses in
separate public offerings. -(Refer to Case Scenario 1)
1. Why do you suppose SI entered the non-retail businesses through acquisition? Is this a cheaper
route than starting up these businesses from scratch?
The best answers may begin by noting that SI had no real prior experience in these non-retail businesses so
they needed to either buy the relevant operations and skills or start them up from scratch. Absent such
experience it is considerably more expedient to enter these businesses through acquisition, since they are likely
to be able to acquire both the business and an experienced management team. The second question gets to the
fact that SI would also likely have to pay a premium for the acquired firms since it brought no industry-specific
knowledge to the bargaining table.
=> I suppose SI entered the non-retail businesses through acquisition is a cheaper route than starting up
these businesses from scratch. The most direct benefits is, it can use the target company's resources, including
human, material, and relationship between all resources, to adjust his capital structure, enterprise structure, so as
to improve performance. And can also be bought with the company had the reputation of the image, brand, to
strengthen our market position, increase operational profits. At the same time, SI can be greatly entered the non-
retail businesses and have time to market through the acquisition. Including the preparation stage, into the stage,
the time required to expand stage. On the other hand, through to the management of the purchase to also can
make SI fast have the normal operation of the function. So I think acquisition is a cheaper route than starting up
these businesses from scratch.
2. Part 1: (Refer to Case Scenario 1) Why do you suppose that SI sold off or spun-off its non-retail
businesses? Part 2: What should SI do after selling off the non-retail businesses?
Part 1: The best answers will note that SI was probably in too many and too many different businesses.
Each of these businesses had to compete in their respective industries while at the same time dealing with SI's
corporate ownership. By getting out of the non-retail business SI is able to get back to its roots in retail. While
of course speculative, students can debate whether or not SI chose the right business to focus its future on. From
a resource-based perspective, retail had the strongest history, which would likely give SI the richest and most
defensible set of valuable, rare, and costly to imitate resources in the retailing business.
Part 2: This is a natural follow-on to Part 1 above. Students could begin this answer by suggesting that SI's
diversification strategy diverted its attention from the needs of its core retailing business. Future efforts should
be directed toward turning the retailing business around and aggressively trying to outmaneuver emerging and
existing retailing competitors. The instructor can use this dialogue to point out that after establishing a strong
industry position, SI probably viewed its retailing business as stable and unthreatened, and thus used it as means
of financing its broad diversification efforts. In contrast, emerging companies like Wal-Mart viewed retailing as
a growth vehicle and developed novel and lower cost structures which eventually undermined the advantages
established earlier by SI.
=> From the paragraph, we can know each of the non-retail businesses of SI was average in its relative
industry performance. SI’s vast mall-based department store holdings were suffering from deferred maintenance
and merchandising that did not appear to be popular with its once large consumer base. So its retail business
basically is not developed. At the same time, in the retail market, high efficiency and low cost on the
competition like Wal-Mart enterprise have more advantage. This is because like Wal-Mart enterprise focusing
only on retail market. And SI would because of other retail industry distracted company's energy. When SI sold
off or spun-off its non-retail businesses, it could focus on company's all resources and take the retail market
share preempted back.
Case Scenario 2: Raptec Raptec was incorporated in 1991 and went public on the Nasdaq Stock
Market in 1996. Raptec's strategy is to become the global leader in innovative storage solutions. Raptec is
an S&P 500 and a Nasdaq Stock Market 100 member. The company's hardware and software solutions
for eBusiness and Internet applications move, manage, and protect critical data and digital content.
Raptec operates in three principal business segments: Direct Attached Storage ("DAS"), Storage
Networking Solutions ("SNS") and Software. These hardware and software products are found in high-
performance networks, servers, workstations, and desktops from the world's leading OEMs, and are sold
through distribution channels to Internet service providers, enterprises, medium and small businesses,
and consumers. Since the time it went public, Raptec has experienced rapid growth and consistently
profitable operations. In early 2007, the company announced its plan to spin-off the software segment,
subsequently incorporated as Axio, Inc., in the form of a fully independent and separate company.
Software was Raptec's most profitable and fastest growing segment. By mid-2007 Raptec had completed
the initial public offering of approximately 15% of Axio's stock, and then distributed the remaining Axio
stock to Raptec's stockholders in a tax-free distribution. -(Refer to Case Scenario 2)
1. Why would a successful firm like Raptec spin off its most promising business?
The best answers will begin by noting that both hardware and software are industries characterized by fast
cycle times, which requires management to be both focused and nimble. With this background, students can
then argue that the spin-off provides the management teams of Raptec and the newly formed Axio with greater
focus (on hardware and software respectively), better alignment of employee incentives, and greater managerial
accountability. The spin-off also provides Axio direct access to capital markets.
2. Prior to the spin-off, how would you go about identifying the respective boundaries of the Raptec
and Axio businesses?
The purpose of this question is to point out how blurry the lines may be between businesses in a diversified
firm - the best answers will revolve around this point. While Raptec operated in three business segments, this
does not guarantee that each operated as independent organizations within Raptec. In fact, Raptec likely
benefited from tremendous operational and market synergies among its three primary lines of business, and such
synergies are typically accomplished through formal coordination and integration mechanisms like
organizational structure, systems, and processes. A useful analogy here can be drawn to Palm, Inc., and its PDA
product the Palm Pilot. Given that consumers view the Palm PDA as a monolithic product (they don't think of it
as separate hardware and software, where would you begin to draw the dividing line if Palm wanted to split up
its hardware and software businesses?
3. What risks does Raptec run in spinning off Axio?
The best answers will point out that the spin-off strategy makes sense only to the extent that the benefits
described in the answer to question 1 considerably outweigh the costs arising from breaking up the firm and its
lost opportunities for within-firm synergies. If Raptec has been successful because of its ability to uniquely
couple hardware and software, along with the fact that it possesses inside knowledge about the technological
advances in each business, then breaking up the firm may actually break-up and destroy a potential core
competency. Also, once a firm has broken itself up into distinct legal entities there is nothing to prevent one of
the players from preying on the others' most profitable related businesses. For instance, Axio may start moving
into parts of the hardware business that, from its inside experience with Raptec, it knows are highly profitable
when combined with Axio's proprietary software.
Case Scenario 3: Barracuda Inc. Barracuda Inc. has diversified beyond its early base as a lamp
fixture manufacturer into multiple hardware and plumbing fixture products that it sells to professionals
(i.e., plumbers and electricians) and through the large volume do-it-yourself (DIY) stores like The Home
Depot and Lowe's. While this successful growth has been achieved primarily through acquisition, the
company tends to let the acquired businesses run independently. It has done so by looking to fragmented
industries to acquire small firms with efficient operations and good management teams. It then grows
these businesses through a combination of internal cash flow and debt, and directs new sales to the
professional and DIY channels. Barracuda has been particularly successful in the faucet segment, which
it practically reinvented though such technological innovations as the washerless faucet, and marketing
innovations like branding and good-better-best merchandising. Barracuda has leveraged this
merchandising strategy across its businesses and, coupled with the explosive growth of the DIY channel,
is spectacularly profitable with a net profit after tax (NPAT) of 18%. The firm's management is looking
to broaden its revenue base and has identified the home furnishings business as sharing many
characteristics with faucets, prior to Barracuda's entry into faucets. It plans to enter this industry
through large-scale acquisitions. The landscape of the U.S. home furnishings manufacturing industry
consists of many players, none with controlling share, and serious issues of overcapacity. There are
presently 2500 home furnishings firms, and only 600 of those have over 15 employees. Average NPAT is
between 4 and 5%, which also reflects the fact that few firms have good managers. While the industry is
still primarily comprised of single-business family-run firms, which manufacture furniture domestically,
imports are increasing at a fairly rapid rate. Some of the European imports are leaders in contemporary
design. Relatively large established firms are also diversifying into the home furnishings industry via
acquisition. Supplier firms to the home furnishings industry are in relatively concentrated industries (like
lumber, steel, and textiles), and therefore typically offer fewer accommodations to the small furniture
manufacturers. Retailers, the intermediate customer of the home furnishings industry, are becoming
increasingly concentrated and the few large, successful furniture companies actually have their own
stores or have dedicated showrooms in the larger department stores. Customers have many products to
choose from, at many different price points, and few home furnishing products beyond those of the larger
companies have established brands. Also, customers can switch easily among high and low-priced
furniture and other discretionary expenditures (spanning plasma TVs to the choice of postponing any
furniture purchase entirely). -(Refer to Case Scenario 3)
https://pdfslide.net/documents/sm5450d1d2b1af9f2c118b4cc2.html
1. Why would Barracuda consider acquisition as its preferred mode of entry into furniture?
Barracuda considers acquisition as its preferred mode of entry into furniture to ensure the immediate
market presence in the furniture business and also to emphasize on the business diversification strategy. As
acquisition is the process to takes over the controlling interest of rivals, so another concern
was to gain market share and competencies, from the previous successful growth was achieved
primarily through acquisition gave them an experience of being successful again by expanding them with
appropriate strategy. Barracuda’s consideration of acquisition is also being helpful to establish a powerful brand
name in the furniture segment, but not forgetting valuable knowledge, skill and experience of the employee
which will transfer into Barracuda from the acquired industry. Distribution channel of the acquiesced
firm is an added advantage to cut or reduce unnecessary cost for Barracuda. From the above argument it is
well understood that the Barracuda considering acquisition is a beneficial steps of the company to enter into
furniture.
2. Given the history of Barracuda, what guidelines would you suggest to management regarding
their acquisition strategy in the home furnishings industry?
Based on the case, Barracuda successful growth has been achieved primarily through acquisition, the
company tends to let the acquired businesses run independently shows a clearer picture of the good and efficient
management of Barracuda. However, there are several guidelines regarding their acquisition strategy in
the home furnishings strategy. First is when organization competes in a no-growth or a slow-growth
industry.In order to acquire small firm with efficient operation and good management team, then start looking to
fragmented industries. Second is when adding new, but related, product would be significantly enhancing
the sales of current product. As early base of Barracuda Inc is lamp fixture manufacture into multiple
hardware and plumbing fixture product and has identified the home furnishing business as shaving
characteristic management team. Third is when new, but related, products could be offered at highly
competitive prices. Barracuda after do an acquisition, they can put the price of higher than their competitors. As
quality of the product are customer focused.
3. Given Barracuda's history, what threats does Barracuda face in entering the furniture industry
through acquisition?
Given Barracuda’s history of having high NPAT in other businesses, ne of the threats of entering the home
furnishing business is low NPAT as historically this industry haslower return due to lack of good management
and operation. Barracuda may not be able to run the home furnishing segment indecently and this
may require high attention from managers to integrate and build the business and it may cause other pertinent
issues to be neglected. Barracuda has been successful with their technological and marketing
innovation but the fact that lower NPAT in the home furnishing segment may lead to shift of focus
from innovation to financial success rather than strategic control of other business. Innovation may be ignored
as Barracuda tries to achieve synergy and improve financial standing of the newly acquired.
4. What is the core business of Barracuda and identify the company strength.
The core business of Barracuda is lamp fixture manufacturer into multiple hardware and plumbing fixture
product that it sells to professional. The strength of Barracuda issuccessful in the faucet
segment, which is practically reinvented through such technological innovation as the washerless
faucet, and marketing innovation like branding and good better best merchandising. Following with next
strength which is strategy across its business and coupled with explosive growth of DIY channel.
5. Describe at least 2 strategies used by Barracuda.
a) Product development: Product development is a strategy that seeks in increased sales by
improving or modifying present products or services. Product development usually entails large research
and development expenditure. As stated, Barracuda is seeking increased sales by improving present on services
or developing as new one.
b) Related diversification: Related diversification is about adding new but related product or services. The
firm management is looking to broaden its revenue base and has identified the home furnishing
business as sharing many characteristic with faucets, prior to Barracuda entry into faucets. It can help the
business will more efficient and good management team.
Quiz 8: International Strategy
http://www.staequiz.com/product/the-management-of-strategy-concepts-international-edition-
10th-edition-by-r-duane-ireland-test-bank/
https://quizlet.com/85345218/strategic-management-ch-8-essay-flash-cards/
https://www.studyblue.com/notes/note/n/chapter-8-international-strategy/deck/7118953
1. What are the motives for firms to pursue international diversification? What are the four basic
benefits firms can derive by moving into international markets?
One reason is to extend the life cycle of the firm's products. Gaining access to needed and potentially
scarce resources is another reason. There is also pressure for global integration of operations, driven by growing
universal product demand. Companies also want to take advantage of opportunities to better use rapidly
developing technologies such as the Internet and mobile applications, which permit greater integration of trade,
capital, and culture. Finally, the potential of large demand for goods and services for people in emerging
markets is an important incentive. When firms successfully move into international markets, they can
experience: increased market size, economies of scale and learning, and location advantages.
2. What are the four factors that provide a basis for international business-level strategies?
Firms derive three basic benefits by successfully using international strategies: (1) increased market size,
(2) economies of scale and learning, and (3) advantages of location. Increased market size is achieved by
expansion beyond the firm's home country. International expansion increases the number of potential customers
a firm may serve. Starbucks is a firm that has increased its market size through international expansion
(Opening Case). Other firms such as Coca Cola and PepsiCo have moved into international markets primarily
because of limited growth opportunities in their domestic markets. Economies of scale and learning is a second
benefit. Leveraging a technology beyond the home country allows for more units to be sold and initial
investments recovered more quickly. Rivals Airbus and Boeing have multiple manufacturing facilities and
outsource some activities in order to gain scale advantages. Lastly, advantages of location can be realized
through internationalization. These advantages include access to low-cost labor, critical resources, or customers.
3. Discuss the three international corporate-level strategies. On what factors are these strategies
based?
International corporate strategy focuses on the scope of a firm's operations through both product and
geographic diversification. The three basic international corporate- level strategies vary on the need for local
responsiveness to the market and the need for global integration. The multidomestic strategy focuses on
competition within each country in which the firm operates. Firms employing a multidomestic strategy
decentralize strategic and operating decisions to the strategic business units operating in each country so
business units can customize their goods and services to the local market. The use of global integration in this
strategy is low. The global strategy assumes more standardization of product demand across country boundaries.
Therefore, competitive strategy is centralized and controlled by the home office, placing high emphasis on
global integration of operations. The strategic business units in each country are interdependent and the home
office integrates these businesses. The firm offers standardized products across country markets. It emphasizes
economies of scale and the opportunity to use innovations developed in one nation to other markets. The
transnational strategy seeks to achieve both global efficiency (through global integration) and local
responsiveness. This strategy is difficult to implement. One goal requires global coordination while the other
requires local flexibility. Flexible coordination builds a shared vision and individual commitment through an
integrated network. The effective implementation of the transnational strategy often produces higher
performance than either of the other corporate-level strategies.
4. Identify and describe the modes of entering international markets. What are their advantages and
disadvantages?
Firms may enter international markets in any of five ways: exporting, licensing, forming strategic alliances,
making acquisitions, and establishing new, wholly owned subsidiaries (greenfield ventures).
● Most firms, particularly small ones, begin with exporting (marketing and distributing their products
abroad). This involves high transportation costs and possibly tariffs. An exporter has less control over the
marketing and distribution of the product than in other methods of entering the international market. In addition,
the exporter must pay the distributor or allow the distributor to add to the product price in order to offset its
costs and earn a profit. In addition, the strength of the dollar against foreign currencies is a constant uncertainty.
But, the advantages are that the company does not have the expense of establishing operations in the host
countries. The Internet makes exporting easier than in previous times.
● Licensing (selling the manufacturing and distribution rights to a foreign firm) is also popular with
smaller firms. The licenser is paid a royalty on each unit sold by the licensee. The licensee takes the risks and
makes the financial investments in manufacturing and distributing the product. It is the least costly way of
entering international markets. It allows a firm to expand returns based on a previous innovation. But there are
disadvantages. Licensing provides the lowest returns, because they must be shared between the licensee and the
licensor. Licensing gives the licenser less control over the manufacturing and marketing process. There is the
risk that the licensee will learn the technology and become a competitor when the original license expires. If the
licenser later wishes to use a different ownership arrangement, the licensing arrangement make create some
inflexibility.
● Strategic alliances involve sharing risks and resources with another firm in the host country. The host
country partner knows the local conditions; the expanding firm has the technology or other capabilities. Both
partners typically enter an alliance in order to learn new capabilities. The partnership allows the entering firm to
gain access to a new market and avoid paying tariffs. The host-country firm gains access to new technology and
innovative products. Equity-based alliances are more likely to produce positive gains. Alliances work best in the
face of high uncertainty and where cooperation is needed between partners and strategic flexibility is important.
But, many alliances fail due to incompatibility and conflict between the partners. Cross- border acquisitions
provide quick access to a new market, but they are expensive and involve complex negotiations. Cross-border
acquisitions have all the problems of domestic acquisitions with the complication of a different culture, legal
system and regulatory requirements. Acquisitions are expensive and usually involve debt- financing.
● The most expensive and risky means of entering a new international market is through the establishment
of a new, wholly owned subsidiary or greenfield venture. Alternatively, it provides the advantages of maximum
control for the firm and, if successful, potentially the greatest returns as well. This alternative is suitable for
firms with strong intangible capabilities and/or proprietary technology. The risks are high because of the
challenges of operating in an unfamiliar environment. The company must build new manufacturing facilities,
establish distribution networks, and learn and implement new marketing strategies.
5. Discuss the effect of international diversification on a firm's returns.
Research shows that returns vary as the level of diversification increases. At first, returns decrease, then as
the firm learns to manage the diversification, returns increase. But, as diversification increases past some point,
returns level off and become negative. Firms that are broadly diversified in international markets usually receive
the most positive stock returns, especially when they diversify geographically into core business areas.
International diversification can lead to economies of scale and experience, location advantages, increased
market size, and the potential to stabilize returns (which reduces the firm's overall risk). International
diversification improves a firm's ability to increase returns from innovation before competitors can overcome
the initial competitive advantage. In addition, as firms move into international markets, they are exposed to new
products and processes that stimulate further innovation. The amount of international diversification that can be
managed varies from firm to firm and according to the abilities of the firm's managers. The problems of central
coordination and integration are mitigated if the firm diversifies into more friendly countries that are
geographically and culturally close.
6. Identify and describe the major risks of international diversification.
International diversification carries multiple risks. The major risks are political and economic. Political
risks are related to governmental instability and to war. Instability in a government creates economic risks and
uncertainty created by government regulation. Governmental instability can result in the existence of many
potentially conflicting legal authorities, corruption, and the risk of nationalization of company assets. Economic
risks are related to political risks. Economic risks include the increased trend of counterfeit products and the
lack of global policing of these products. Another economic risk is the perceived security risk of a foreign firm
acquiring firms that have key natural resources or firms that may be considered strategic in regard to intellectual
property. In addition, differences in and fluctuations of the value of different currencies is another economic
risk. The security risk created by terrorism prevents U.S. firms from investing in some regions. The relative
strength or weakness of the dollar affects international firms' competitiveness in certain markets and their
returns.
Case Scenario 1: Blast Furnace, Inc., (BFI) Blast Furnace, Inc., (BFI) provides customized
development of automated rich-media applications, and scalable solutions that allow media and
entertainment companies, as well as enterprises and government organizations, to deploy, manage and
distribute video content on IP-based networks. The company was founded in 1997 and went public in
2004; its stock trades on the NASDAQ under the ticker BLST. While providing solutions to a variety of
firms and industries in North America, BFI has experienced its fastest growth with the security products
that it designs and sells to the U.S. government and U.S. government agencies. This growth is based on its
propriety VUE software, which is a complete identification solution for capturing, analyzing, and
managing multi-biometric information. Proprietary analysis algorithms aggregate and cross-compare
multiple biometrics to increase accuracy and lessen dependence on single identification techniques.
Additionally, specialized encoding techniques reduce file size and increase analysis and response times.
VUE supports a wide range of applications ranging from ID issuance and verification to gated entry
screening for border patrol, airports, government buildings, and corporations. Essentially, VUE is able to
sift through massive amounts of digitized multimedia files to create a unified ID dossier of an individual
and then identify those individuals rapidly anywhere in the data. Such a capability is of great interest to
security organizations, particularly since the World Trade Center bombing, because it allows the user to
identify suspects within minutes on a real-time basis anywhere digitized media is being created (like that
created by the hidden and visible surveillance cameras in airline terminals, banks, ATMs, and other
public locations). Once a suspect is entered into the system, the software is capable of scanning all data
sources automatically and without stop. For two years, BFI has had this market to itself but now two new
entrants, a Belgian start-up and the subsidiary of a Finnish telecom firm, are staking out positions in
large non-U.S. markets like Europe and Asia. BFI's management fears that if it limits its efforts to North
America, then these aggressive competitors may eventually develop strongholds in other markets from
which they can launch successful attacks on BFI's home turf. -(Refer to Case Scenario 1)
1. Should BFI expand its operations outside of North America?
2. Assume that BFI has chosen international expansion. How quickly should it move? Which activities
would you recommend BFI to internationalize first?
3. How would you evaluate which country or countries BFI should enter first?
Case Scenario 2: Heartsong LLC. Heartsong LLC is a designer and manufacturer of replacement
heart valves based in Peoria, Illinois. While a relatively small company in the medical devices field, it has
established a worldwide reputation as the provider of choice of high-quality, leading-edge artificial heart
valves. Most of its products are sold to large regional hospital systems and research hospitals around the
world, though primarily to customers in the U.S. and Europe. Specialty heart centers are another
emerging, but fast-growing market for its valves. Heartsong has recently embarked on an expansion
strategy that requires it to increase its volume, which in turn will demand more component parts than it
can source domestically - both from an economic and volume standpoint. The firm has determined that
such growth is only viable if it produces these parts itself overseas for a lower cost, or outsources the
production entirely to a joint venture it establishes with a local manufacturer, which could both produce
the parts more cheaply and in higher volumes. It is considering starting up an owned production facility
in Luxembourg, or seeking a joint venture with a precision manufacturer in China. -(Refer to Case
Scenario 2)
http://www.ziwojieshao.org/fanwen/359912.html
https://issuu.com/mcdonaldmcdonald/docs/strategic-management-concepts-and-c
1. What opportunities and threats might Heartsong be exposing itself to via the Luxembourg
expansion proposal?
The best answers will begin by noting that Heartsong has the capacity to design leading-edge medical
products and then take these designs and turn them into reliably manufactured, high-quality replacement heart
valves. Thus, basic R&D and quality precision manufacturing are likely to be critical value-creating facets for
this firm.
2. What opportunities and threats might Heartsong be exposing itself to via the China expansion
proposal?
The best answers will start by observing that the scenario suggests that Heartsong needs to grow if it is
going to continue being competitive and successful. However, Heartsong is also capital constrained and an
outsourcing arrangement with EdFex allows it to more efficiently manage this significant aspect of its cost base
(inventory and delivery). This outsourcing solution would be ideal if it would allow Heartsong to maintain a
centralized warehouse with heart valve inventory in major population centers, instead of its present practice of
carrying inventory on the shelves of each of its hospital customers. As a result, Heartsong could grow its market
presence, while more efficiently managing the need to have heart valves available on short notice.
3. Which option would you recommend?
The best answers will develop the theme that the EdFex outsourcing arrangement is truly likely to be win-
win. With the arrangement in place, Heartsong is able to devote its financial, human capital, and managerial
resources to basic R&D and quality precision manufacturing; and, EdFex does what it does best in logistics.
Moreover, it is hard to contemplate that EdFex would ever think of entering the heart valve industry – thus,
EdFex does not pose a direct threat as a future competitor. It does however pose an indirect threat to Heartsong
to the extent it can hold the firm hostage, and extract exorbitant fees for its logistic services.
Quiz 9: Cooperative Strategy
https://quizlet.com/145599064/final-part-2-multiple-choice-only-9-flash-cards/
1. Identify and define the different types of strategic alliances.
Strategic alliances are cooperative strategies between firms whereby resources and capabilities are
combined to create a competitive advantage. All strategic alliances require firms to exchange and share
resources and capabilities to co-develop or distribute goods or services. The three basic types of strategic
alliances are: (1) joint ventures, where a legally independent company is created by at least two other firms,
with each firm usually owning an equal percentage of the new company; 2) equity strategic alliances, whereby
partners own different percentages of equity in the new company they have formed; and (3) non equity strategic
alliances, which are contractual relationships between firms to share some of their resources and capabilities.
The firms do not establish a separate organization, nor do they take an equity position. Because of this, non
equity strategic alliances are less formal and demand fewer partner commitments than joint ventures and equity
strategic alliances. Typical forms are licensing agreements, distribution agreements and supply contracts.
2. Explain the rationales for a cooperative strategy under each of the three types of basic market
situations (i.e., slow, standard, and fast cycles).
In slow-cycle markets (markets that are near-monopolies), firms cooperate with others to gain entry into
restricted markets or to establish franchises in new markets. Slow-cycle markets are rare and diminishing.
Cooperative strategies can help firms in (presently) slow-cycle markets make the transition from this relatively
sheltered existence to a more competitive environment. In standard-cycle markets (which are often large and
oriented toward economies of scale), firms try to gain access to partners with complementary resources and
capabilities. Through the alliance, the firms try to increase economies of scale and market power. In fast-cycle
markets (characterized by instability, unpredictability, and complexity) sustained competitive advantages are
rare, so firms must constantly seek new sources of competitive advantage. In fast-cycle markets alliances
between firms with excess resources and capabilities and firms with promising capabilities who lack resources
help both firms to rapidly enter new markets.
3. Identify the four types of business-level cooperative strategies and the advantages and
disadvantages of each.
Through vertical and horizontal complementary alliances, companies combine their resources and
capabilities in ways that create value. Vertical complementary strategic alliances result when firms creating
value in different parts of the value chain combine their assets to create a competitive advantage. Vertical
complementary strategies have the greatest probability of being successful compared with other types of
cooperative strategies. But firms using this type of alliance need to be wise in how much technology they share
with their partners. Vertical complementary alliances rely heavily on trust between partners to succeed.
Horizontal complementary strategic alliances are developed when firms in the same stage of the value chain
combine their assets to create additional value. Usually they are formed to improve long-term product
development and distribution opportunities. Horizontal complementary strategies can be unstable because they
often join highly rivalrous competitors. In addition, even though partners may make similar investments, they
rarely benefit equally from the alliance. The competition response strategy involves alliances formed to react to
competitors' actions. Usually they respond to strategic, rather than tactical, actions because the alliances are
difficult to reverse and expensive to operate. The uncertainty-reducing strategy is used to hedge against risk and
uncertainty, such as when entering new product markets or in emerging economies. Both of these strategies are
less effective in the long-run than the complementary alliances which are focused on creating value.
Competition reducing (collusive) strategies are often illegal. There are two types of collusive competition
reducing strategies: explicit collusion and tacit collusion. Explicit collusion exists when firms directly negotiate
production output and pricing agreements to reduce competition. These are illegal in the U.S. and in most
developed economies. Tacit collusion exists when several firms in an industry indirectly coordinate their
production and pricing decisions by observing each other's competitive actions and responses. Both types of
collusion result in lower production levels and higher prices for consumers.
4. Identify the three types of corporate-level cooperative strategies.
A diversifying strategic alliance allows firms to share some of their resources and capabilities to diversify
into new product or market areas. A synergistic strategic alliance allows firms to share some of their resources
and capabilities to create economies of scope. These alliances create synergy across multiple functions or
multiple businesses between partner firms. Franchising is a strategy in which the franchisor uses a contractual
relationship to describe and control the sharing of its resources and capabilities with franchisees. A franchise is
a contract between two independent organizations whereby the franchisor grants the right to the franchisee to
sell the franchisor's product or do business under its trademarks in a given location for a specified period of
time.
5. Why are cooperative strategies often used when firms pursue international strategies? What are
the advantages and disadvantages of international cooperative strategies?
A cross-border strategic alliance is an international cooperative strategy in which firms headquartered in
different nations combine some of their resources and capabilities to create a competitive advantage. The typical
reasons follow: 1) In general multinational firms outperform firms operating only on a domestic basis. Firms
may be able to leverage core competencies developed domestically in other countries. 2) Limited domestic
growth opportunities pushes firms into international expansion. 3) Some governments require local ownership
in order for foreign firms to invest in businesses in their countries, which requires foreign firms to ally with
local firms. 4) Local partners often have significantly more information about factors contributing to
competitive success such as local markets, sources of capital, legal procedures, and politics, which makes an
alliance useful for a foreign firm. 5) Cross-border alliances can help firms transform themselves or better use
their competitive advantages surfacing in the global economy. On the negative side, cross-border alliances are
more complex and risky than domestic strategic alliances.
6. Identify and define the two different types of network strategies.
A network cooperative strategy is a cooperative strategy wherein several firms form multiple partnerships
to achieve shared objectives. Stable alliance networks (primarily found in mature industries) usually involve
exploitation of economies of scale or scope. In this type of network, the firms try to extend their competitive
advantages to other settings while continuing to profit from operations in their core industries. Dynamic alliance
networks (witnessed mainly in rapidly changing industries) are used to help a firm keep up when technologies
shift rapidly by stimulating product innovation and successful market entries. Dynamic alliance networks
explore new ideas and typically generate frequent product innovations with short product life cycles.
7. Identify the competitive risks associated with cooperative strategies.
Cooperative strategies are not risk free strategy choices; as many as 70% fail. If a contract is not developed
appropriately and fails to avert opportunistic behavior, or if a potential partner firm misrepresents its
competencies or fails to make available promised complementary resources, failure is likely. Furthermore, a
firm may make investments that are specific to the alliance while the partner does not. This puts the investing
firm at a disadvantage in terms of return on investment. The core of many failures is the lack of trustworthiness
of the partner(s) who act opportunistically.
8. Describe the strategic management approaches to managing alliances.
The ability to effectively manage competitive strategies can be one of a firm's core competencies. There are
two basic approaches to managing competitive alliances. Cost minimization leads firms to develop protective
formal contracts and effective monitoring systems to manage alliances. Its focus is to prevent opportunistic
behavior by the partner(s). Opportunity maximization is intended to maximize value creation opportunities. It is
less formal and places fewer constraints on partner behaviors. But, identifying trustworthy partners is the key to
this second approach. If (well-founded) trust is present, monitoring costs are lowered and opportunities will be
maximized. Trust is more difficult to establish between international partners. Ironically, the cost minimization
approach is more expensive to implement and to use than the opportunity maximization approach.
Case Scenario 1: Norning International Norning International (NI) states that both its past successes
and future growth strategies are based on an evolving network of wholly owned businesses and joint
ventures around its core competency in glass making. Through their alliances and owned divisions they
compete in four global business sectors: Specialty Glass and Materials (including materials for HDTV
and LCD displays), Consumer Housewares (including microwavable dishware), Laboratory Sciences
Products and Services (test tubes, testing equipment, and drug trials testing), and Communications (fiber
optics and related technologies). Per the company's annual report, "binding all four sectors together is
the glue of a commitment to leading edge glass making technologies, shared resources, and dedication to
total quality." Each sector is composed of divisions, subsidiaries and alliances. However, the central role
played by alliances is demonstrated by the fact that the combined revenue of its 30-some alliances is more
than double that of NI on its own. Most of the alliances provide NI with access to particular geographic
markets, industries, or channels, although an increasing number of alliances involve both market access
and technological development. -(Refer to Case Scenario 1)
1. Why would a company like NI place such emphasis on alliances as a growth vehicle?
There are many components that are involved in glassmaking technologies, so in order to gain more
resources and produce a better product to consumers, it is essential to form alliances to merge their different
segments for efficiently. Also, for firms like NI who don't have enough capital to acquire a firm, they can form a
less capital intensive agreement to partner with another firm.
Moreover, if NI is able to retain its focus and leading edge expertise in advanced glass-making techno-
logies, while at the same time realize repeated successes and technological breakthroughs via alliances, then its
ability to partner and manage multiple alliances may truly become a strategic asset.
2. What risks arise from a strategy based on such a "network of alliances"?
The best answers will start by noting that NI’s historic success (via its core competency in glass making) is
likely to have been based in a strong commitment to basic research and development. Multiple partnerships, in
and of themselves, can lead to the dilution of management's attention and investment in this core, or lead to a
less well defined core. Given that such basic R&D is the “glue” holding NI together, such dilution may make
that glue at the hub of its strategy wheel come unstuck. Exacerbating this threat is the possibility that NI’s
fascination with alliances may become a crutch where alliances displace its internal strengths and become an
end in and of themselves. Alliance partners are also a threat to the extent they are able to learn more from NI
about its technologies, and eventually undermine or leapfrog NI technological competitive advantages.
3. NI appears to be managing a large number of alliances. What criteria should it use to exit
particular alliances?
The best answers will develop a set of company rules of thumb for exiting alliances, and even some of NI’s
owned businesses. For instance, given the fundamental role played by the need to emphasize ba-sic R&D in
advanced glass-making technologies, students may first start by observing that NI should likely exit those
businesses that are highly service-based (like some of its laboratory services businesses), or do not require
leading edge glass-making expertise (like consumer glass-based products), since neither of these businesses
require or contribute to NI’s understanding of new glass-making technologies. Consequently, NI should exit the
alliances related to those particular segments because they don’t help to reinvigorate its technological core.
Beyond this rule of thumb, students may then push NI to exit those alliances where there is the least opportunity
to learn from its partners, and again reinvigorate and further enhance its core in advanced glass making.
Case Scenario 2: ERP Inc ERP, Inc., (ERPI) is a leading provider of enterprise integration software
(EIS). EIS essentially allows a firm to connect and integrate processes across all aspects of its business. To
fuel its dramatic growth, ERPI has focused its organization entirely on product development (software
programming for a suite of EIS products) and selling (making the sale and then moving onto a new
target), while outsourcing the installation and consulting aspects to the world's largest accounting firms.
This also makes ERPI basically a "product company," whereas most competitors like Oracle and
PeopleSoft in its market space operate as "solutions companies." One benefit of this focused strategy is
that ERPI's product is generally recognized as being 200% to 300% better than competitors' software,
and thus adopters are thus likely to have a one to two year advantage. In further contrast to the
competition, ERPI has used its partnerships with the accounting firms to deliver a turn-key solution, and
has focused this solution on a market comprised of the world's largest, global manufacturers and
consumer product companies. The accounting firms, in turn, coordinate a comprehensive collection of
hardware, operating systems, and complementary software firms. Installation and related consulting for
EIS typically cost between $100 and $200 million, with the ERPI software component accounting for only
about 20% of the installed cost (the remaining 80% is spent on the actual installation, not counting the
value of the customer's time). To incentivize the accounting firms to help sell its product (since, at least
initially, the accounting firms had better reputations and controlled access to the target customers), ERPI
told its partners that it will never enter the installations and consulting side of the business (aside from
installation and consulting that ERPI does as part of its software support). Dangling such a large carrot
in front of the accounting firms provided the continuing benefit of encouraging their continued support of
ERPI with their customers. -(Refer to Case Scenario 2)
1. Given that software systems like EIS are very complex, and quality is largely a function of the
related installation and consulting processes, how can ERPI control quality and ultimately protect the
reputation of its product (and its name) when it has ultimately outsourced installation to its partners?
The best answers will simply walk through the respective columns in Table 3.5 and reach the conclusion
that, at least in the near term, ERPI has a sustainable competitive advantage. Its EIS software is valuable given
that it is 200% to 300% better than competitors’ products. It is similarly rare and nonsubstitutable since it is
proprietary, and currently has a two-year lead on the alternatives. A similar rationale can be invoked to support
the argument that ERPI’s capabilities in software programming are going to be costly to imitate. A competitor
would have to hire a similar workforce or acquire a company that currently occupies the same market space.
This strong position is further bolstered by the fact that a large percentage of the market is voting with its feet in
favor of ERPI.
2. After managing this network of alliances for several years, what new strategic assets has ERPI
developed?
This question asks students to take a more dynamic perspective of potentially valuable resources that
companies and their customers create together, but that the company itself can exploit (a perfect example of a
co-specialized asset) The best answers will begin by observing that if ERPI has focused histonically on
transactions (making the sale), then it has given hittle explicit consideration to customers as long-term
relationships beyond the need to provide technical support (lifetime value of a customer beyond the first sale).
Shifting attention to ERPI installations as relationships suggests that the company now has a customer list to die
for. This list is especially valuable since (1) the target companies have invested upwards of $200 million in
ERPI proprietary systems and, (2) once installed, given the pervasive nature of EIS systems, those target firms
are unlikely to simply switch to another system.
3. Imagine that ERPI has saturated the large-firm market for its products, competitors are
undermining its technological advantage, and ERPI needs to look to new markets for revenue. Its CEO
has suggested that it start selling its software down-market to middle-market companies, and at the same
time enter the consulting and installation side of the business for this target market. What are the risks
and opportunities of such a strategy?
The best answers will build on the basic notions developed in response to question 4., Students will argue
that ERPI's competitive advantage is sustainable as long as its technology continues to define the leading edge
of EIS products and that substitute solutions do not encroach much on its two-vear lead. However, and as is
consistent with most high-technology markets, as students pick apart ERPI's capabilities following the
categories in Table 3.5 they should begin to see that sustained competitive advantage in this particular market
space may be difficult, particularly given the presence of large aggressive competitors like Oracle and
Microsoft, which are intent on gaining a presence in the EIS market
Case Scenario 3: Bunnywac. Bunnywac is a global producer and seller of batteries for consumer
electronics products (radios, flashlights, toys, etc.), and competes primarily with its larger rivals by
providing battery products equal in performance at a lower price. The worldwide battery industry
suffers from issues of overcapacity and commoditization, brand segmentation and proliferation, the
growing strength of global retailers, and the low-cost threat of new entrants from Asia. Thus, the ability
to provide dependable batteries at a very low cost is essential to survival in this industry. Bunnywac has
grown quickly into one of the leading players in the battery industry primary through horizontal
acquisitions financed by a recent successful IPO, and is now counted among the top four companies in
North and Latin America. Its presence in Europe and Latin America is negligible. While its market
presence and brand is generally strong and market share is growing, Bunnywac has entered into an
alliance to obtain the core technologies of its batteries. Bunnywac does not actually own the technology
that makes its batteries work. This approach has provided Bunnywac a cost advantage since it has not
had to invest in basic R&D and has very little R&D infrastructure. This technology is licensed from Mats
(which has 200 engineers dedicated to moving the technology forward), one of Japan's largest technology-
based holding companies (like Sharp or Canon). Mats also sells batteries under the Pandemonium brand
and commands over 50% of the market share of Asian countries. Mats' market share in other global
markets is negligible and its efforts at growing its branded battery share in the North America, Latin
America, and Europe has been severely frustrated in recent years. While Mats is very large compared to
Bunnywac, the battery technology and battery business are relatively tiny relative to Mats' other
technology-based businesses. Bunnywac's decade-long licensing agreement with Mats for the essential
battery technology expires in one year; there are no obvious substitute providers of this technology. -
(Refer to Case Scenario 3)
1. What should be Bunnywac's primary concerns about its lapsing technology contract with Mats?
The best discussions will draw out near and longer-term concerns for Bunnywac’s prospects, as well as
speculation as to Mats’ long-term strategic intent. Near-term, Bunnywac needs to have access to a reasonably
priced technology if it is to continue competing on a low-cost basis. Also, since it is a public company, the
capital markets are likely to begin showing concern for Bunnywac’s prospects if no replacement technologies
are identified, which in turn will hamper the firm’s ability to grow further through acquisition. Medium-term, if
a new contract is negotiated, Bunnywac needs to be concerned with the price it will now pay, since Mats can be
construed to be in the more advantageous bargaining position. If Bunnywac is determined to remain
independent, then it should only pay such higher costs if it has a parallel near-term strategy of developing its
own technological competencies (which are admittedly very hard and costly to develop) or another source for
the technology at a lower price and on a long-term basis. Finally (and perhaps first!), Bunnywac needs to assess
Mats’ strategic intent with regard to Bunnywac. Does Mats view, and is hence positioning for, Bunnywac as (a)
a valued-long term partner, (b) a potential future competitor, or (c) a near-term cheap acquisition target as a
vehicle for gaining a quick foothold in the branded battery markets?
2. What should Bunnywac's strategy be with regard to the lapsing technology contract?
Following the discussion of its concerns, the best answers will have several possible steps. The first step
should be for Bunnywac to determine Mats’ long-term strategic intent. Is Mats primarily interested in retaining
the technology? Would they sell the technology outright? Is their primary intent to forward integrate into all
global markets through their own or acquired brands? These answers determine whether or not Mats’ intent is to
see Bunnywac as a viable, independent avenue for its technologies, or simply as a potential future competitor, or
something in between. For instance, if Mats sees Bunnywac as adding value in terms of brand and channel
management capabilities, but is not interested in extending the technology alliance, then perhaps Bunnywac
may actually consider negotiating its own sale to Mats while its value is still relatively high.
Quiz 10: Corporate Governance
https://www.studocu.com/en-us/document/rutgers-university/financial-analysis-and-corporate-
governance/past-exams/corporate-governance-chapter-10-exam-answers/2268990/view
1. What is corporate governance and how is it used to monitor and control managers' decisions?
Corporate governance is the relationship among stakeholders that is used to determine and control the
firm's strategic direction and its performance. Effective governance that aligns top-level managers' interests with
shareholders' interests can produce a competitive advantage for the firm. Corporate governance includes
oversight in areas where there are conflicts of interest among major stakeholders, including the election of
directors, supervision of CEO pay, and the organization's overall structure and strategic direction. Three internal
governance mechanisms (ownership concentration, the board of directors, and executive compensation) and an
external mechanism (the market for corporate control) are used in U.S. corporations. Unfortunately, corporate
governance mechanisms are not always successful.
2. Discuss the effect of the separation of ownership and control in the modern corporation.
Ownership is typically separated from control in the large U.S. corporation. Owners (principals) hire
managers (agents) to make decisions that maximize the value of their firm. As risk specialists, owners diversify
their risk by investing in an array of corporations. As decision-making specialists, top executives are expected
by owners to make decisions that will result in earning above-average returns for which they are compensated.
Thus, the typical corporation is characterized by an agency relationship that is created when one party (the
firm's owners) hires and pays another party (top executives) to use decision-making skills. Since owners may
not possess the specialized skill to run a large company, delegating these tasks to managers should produce
higher returns for owners.
3. Define the agency relationship and managerial opportunism and discuss their strategic
implications.
The separation of owners and managers creates an agency relationship. An agency relationship exists when
a principal hires an agent as a decision-making specialist to perform a service. Some problems that result from
the agency relationship between owners and managers include the potential for a divergence of interests and a
lack of direct control of the firm by shareholders. Managerial opportunism is the seeking of self-interest with
guile. It is both an attitude and a set of behaviors, which cannot be perfectly predicted from the agent's
reputation. Top executives may make strategic decisions that maximize their personal welfare and minimize
their personal risk, such as excessive product diversification. Decisions such as these prevent the maximization
of shareholder wealth, which is supposed to be the top executives' priority. Although shareholders implement
corporate governance mechanisms to protect themselves from managerial opportunism, these mechanisms are
imperfect. Agency costs include the costs of managerial incentives, monitoring costs, enforcement costs, and the
individual financial losses incurred by principals (owners of the firm) because governance mechanisms cannot
guarantee total compliance by the agents (managers).
4. Define the three internal corporate governance mechanisms and how they may be used to control
and monitor managerial decisions.
The three internal corporate governance mechanisms are: ownership concentration, the board of directors,
and executive compensation. Ownership concentration is based on the number of large-block shareholders and
the percentage of shares they own. With significant ownership percentage, institutional investors, such as
mutual funds and pension funds, are often able to influence top executives' strategic decisions and actions. Thus,
unlike diffuse ownership, which tends to result in relatively weak monitoring and control of managerial
decisions, concentrated ownership produces more active and effective monitoring of top executives. An
increasingly powerful force in corporate America, institutional owners are actively using their positions of
concentrated ownership in individual companies to force managers and boards of directors to make decisions
that maximize a firm's value. These owners (e.g., CalPERS) have caused poorly performing CEOs to be ousted
from the firm. The board of directors, elected by shareholders, is composed of insiders, related outsiders, and
outsiders. The board of directors is a governance mechanism shareholders expect to run the firm in such a ways
as to maximize shareholder wealth. Outside directors are expected to be more independent of a firm's top
executives than are those who hold top management positions within the firm. A board with a significant
percentage of insiders tends to be weak in monitoring and controlling management decisions. Boards of
directors have been criticized for being ineffective, and there is a movement to more formally evaluate the
performance of boards and their individual members. Executive compensation is a highly visible and often
criticized governance mechanism. Salary, bonuses, and long-term incentives such as stock options are intended
to reward top executives for aligning their goals with the interests of shareholders. A firm's board of directors
has the responsibility of determining the degree to which executive compensation succeeds in controlling
managerial behavior. But, it is difficult to evaluate top executives' performance, andso executive compensation
tends to be linked to financial measures which do not necessarily reflect the effectiveness of the executive's
decision on long-term shareholder outcomes. In addition, many external factors affect the performance of a
firm. Moreover, performance incentive plans can be subject to management manipulation. Consequently,
executive compensation is a far-from-perfect governance mechanism.
5. Discuss the difficulties in establishing performance-based compensation plans for executives.
Executive compensation, especially long-term incentive compensation, is complicated.First, the strategic
decisions made by top-level managers are typically complex and nonroutine; as such, direct supervision of
executives is inappropriate for judging the quality of their decisions. Because of this, there is a tendency to link
the compensation of top-level managers to measurable outcomes such as financial performance.Second, an
executive's decision often affects a firm's financial outcomes over an extended period of time, making it difficult
to assess the effect of current decisions on the corporation's performance. In fact, strategic decisions are more
likely to have long-term, rather than short-term, effects on a company's strategic outcomes. Third, a number of
other factors affect firm performance. Unpredictable economic, social, or legal changes make it difficult to
discern the effects of strategic decisions. Thus, although performance-based compensation may provide
incentives to managers to make decisions that best serve shareholders' interests, such compensation plans alone
are imperfect in their ability to monitor and control managers.Although incentive compensation plans may
increase firm value in line with shareholder expectations, they are subject to managerial manipulation. For
instance, annual bonuses may provide incentives to pursue short-run objectives at the expense of the firm's long-
term interests. Supporting this conclusion, some research has found that bonuses based on annual performance
were negatively related to investments in R&D, which may affect the firm's long-term strategic competitiveness.
Although long-term performance-based incentives may reduce the temptation to underinvest in the short run,
they increase executive exposure to risks associated with uncontrollable events, such as market fluctuations and
industry decline. Long-term incentives may not be highly valued by a manager: thus, firms may have to
overcompensate managers when they use long-term incentives.
6. Describe the market for corporate control and its implications for organizations.
The market for corporate control is composed of individuals and firms who buy ownership positions in
(e.g., take over) potentially undervalued firms to form a new division in an established firm or to merge the two
previously separate firms. The target firm's top management team is usually replaced because it is assumed to be
partly responsible for formulating and implementing the strategy that led to poor firm performance. The market
for corporate control is (supposedly) triggered by low corporate performance by a firm relative to competitors in
its industry. Thus, the market for corporate control should act as a control mechanism for corporate governance
that leads to the replacement of under-performing executives. But, the market for corporate control is not an
efficient governance mechanism because in reality many of the firms taken over have above-average
performance. Hostile takeovers, on the other hand, are typically triggered by poor performance. Some managers
have sought to buffer themselves from the effect of the market for corporate control (hostile takeovers) by
instituting golden parachutes that will pay the managers significant extra compensation if the firm is taken over.
Those and other takeover defenses are intended to increase the costs of mounting a takeover and reducing the
managers' risk of losing their jobs. Examples of takeover defenses include asset restructuring, changes in the
financial structure of the firm, reincorporation in another state, and greenmail. These defense tactics are
controversial and the research on their effectiveness is inconclusive. Most institutional investors oppose them.
7. Briefly compare and contrast corporate governance in the U.S., Germany, and Japan, and China.
Corporate governance structures used in Germany and Japan differ from each other and from the ones used
in the United States. Historically, the U.S. governance structure has focused on maximizing shareholder value.
Banks have been at the center of the German corporate governance structure, because as lenders, banks become
major shareholders in the firms. Shareholders usually allow the banks to vote their ownership positions, so
banks have majority positions in many German firms. The German system has other unique features. For
example, German firms with more than 2,000 employees are required to have a two-tier board structure,
separating the board's management supervision function from other duties that it would normally perform in the
United States (e.g., nominating new board members). Historically, German executives have not been dedicated
to the maximization of shareholder value, because private shareholders rarely have major ownership in German
firms, nor do larger institutional investors play a significant role.
Attitudes toward corporate governance in Japan are affected by the concepts of obligation, family, and
consensus. Japan continues to follow a bank-based financial and corporate governance structure compared to the
market-based financial and corporate governance structure in the United States. In addition, Japanese firms
belong to keiretsu, groups of firms tied together by cross-shareholding. In many cases, the main-bank
relationship of the firm is part of a keiretsu. However, the influence of banks in monitoring and controlling
managerial behavior and firm outcomes is beginning to lessen and a minor market for corporate control is
emerging.Chinese corporate governance has become stronger in recent years. There has been a decline in equity
held in state-owned enterprises, but the state still dominates the strategies employed by most firms. Firms with
higher state ownership tend to have lower market value and more volatility in those values over time. In a broad
sense, the Chinese governance system has been moving toward the Western model in recent years. For example,
YCT International recently announced that it was strengthening its corporate governance with the establishment
of an audit committee within its board of directors, and appointing three new independent directors. In addition,
recent research shows that the compensation of top executives in Chinese companies is closely related to prior
and current financial performance of the firm.
8. How does corporate governance foster ethical strategic decisions and how important is this to top-
level executives?
Governance mechanisms focus on the control of managerial decisions to ensure that the interest of
shareholders, the most important stakeholder, will be served. But shareholders are just one stakeholder along
with product market stakeholders (e.g., customers, suppliers, and host communities) and organizational
stakeholders (e.g., managerial and nonmanagerial employees). These stakeholders are important as well.
Therefore, at least the minimal interests or needs of all stakeholders must be satisfied through the firm's actions.
Otherwise, dissatisfied stakeholders will withdraw their support from one firm and provide it to another (e.g.,
employees will exit and seek another employer, customers seek other vendors, etc.). Some believe that ethically
responsible companies design and use governance mechanisms to ensure that the interests of all stakeholders are
served.Top-level executives are monitored by the board of directors. All corporate stakeholders are vulnerable
to unethical behaviors by the firm. If the image of the firm is tarnished, the image of customers, suppliers,
shareholders, and board members is also tarnished. Top-level managers, as the agents who have been hired to
make decisions that are in shareholders' best interests, are ultimately responsible for the development and
support of an organizational culture that allows unethical decisions and behaviors. The board of directors has the
power and responsibility to enforce this expectation.The decisions and actions of a corporation's board of
directors can be an effective deterrent to unethical behaviors. The board has the power to hold top managers
accountable for unethical actions as they can hire and fire these managers. Thus, the board of directors, which
holds a position above the firm's highest-level managers, holds considerable power over top-level executives
and can set and enforce standards for ethical behaviors within the organization.
Case Scenario 1: Abramson's Jewelers. Abramson's Jewelers has established a strong niche market
in the upscale jewelry store segment. Abramson's was founded in 1871, and its current single-store
location is owned and operated by John Wickersham, who bought the firm from its namesake founders in
1985. Over the last 15 years, Mr. Wickersham has narrowed the company's product offering considerably
to focus only on high-end watches like Rolex and Piaget, custom jewelry, and estate jewelry. Mr.
Wickersham stresses that this is an appropriate focus for his business since each of the products lends
itself to relationship selling, and price rarely comes into the discussion. Despite the narrower offering,
Abramson's floor space has doubled, and clients are intensely loyal to the good taste, design skills, and
personal service level provided by Mr. Wickersham. After evaluating several expansion options, Mr.
Wickersham has decided to open another store in a neighboring city. While it is likely that some of his
existing customers may begin doing business at the other location, thus lowering sales volume at the
original store, Mr. Wickersham sees this as a desirable increase in the level of service and convenience he
can provide his existing clientele. At the same time, he believes that he will be able to grow the overall
business faster with two locations. He has identified another reputable gemologist, Jill Diamond, to run
the other store and is now considering how to compensate her. -(Refer to Case Scenario 1)
1. What are the advantages and disadvantages of paying the new manager primarily cash pay?
The best answers will note, assuming Ms. Diamond has an interest in seeing the store do well, that cashpay
may let her focus on maximizing the level of service provided to the extant customer base. In terms of
drawbacks, there is little incentive for Ms. Diamond to grow the business, which is what Mr. Wickersham is
aiming for.
2. What are the advantages and disadvantages of paying the new manager primarily on new store
sales growth?
The best answers here will contrast the benefits of cash to incentive pay, and note that incentive pay will
focus Ms. Diamond's efforts on increasing the customer base and the dollar sales per customer. A potential
negative consequence would be for her to lose sight of customer service (retention), since she is working so hard
to generate new sales.
3. What compensation structure would you recommend?
Answers should begin by suggesting that both components of pay are important and that some balance
needs to be struck between the two. A fair level of cash pay can motivate Ms.Diamond to keep the existing
business running at a high service level for existing customers. An added incentive pay component can provide
her with additional motivation to grow the business. The best answers here will take the concept of incentive
pay further to suggest that Mr. Wickersham can establish a number of objectives for which the incentive pay can
be earned. Such a balanced scorecard approach provides Ms. Diamond with the incentive to manage the whole
store, and not one or two particular dimensions of it.
Case Scenario 2: Raptec. Raptec operates in three principal business segments: Direct Attached
Storage ("DAS"), Storage Networking Solutions ("SNS") and Software. These hardware and software
products are found in high-performance networks, servers, workstations and desktops from the world's
leading OEMs, and are sold through distribution channels to Internet service providers, enterprises,
medium and small businesses and consumers. Since the time it went public, Raptec has experienced rapid
growth and consistently profitable operations. In early 2002, Raptec announced its plan to spin-off the
software segment, subsequently incorporated as Axio, Inc., in the form of a fully independent and
separate company. Software was Raptec's most profitable and fastest growing segment. By mid-2002
Raptec had completed the initial public offering of approximately 15% of Axio's stock, and then
distributed the remaining Axio stock to Raptec's stockholders in a tax-free distribution. Axio's family of
products includes category leaders in CD/DVD burning, digital photography, and digital video. Axio's
new management team is comprised of: Lex Luthor, CEO, and previously the President of New Business
Development for Universal Studios Recreation Group; Karal Kool, COO, and previously General
Manager of Raptec's OEM Solutions Group; and R. Elliot Maxter, CFO, and previously corporate
controller for Raptec. The interim four-member board of directors is currently comprised of Raptec
senior officers, but the terms of the public offering require them to step down in two months. Thus, Axio
will need to construct a new board, which in turn will be responsible for overseeing Axio's management
and their compensation. -(Refer to Case Scenario 2)
1. What characteristics will you look for in appointing new board members?
The best answers will begin by noting that Axio should probably look outside the firm for new directors,to
provide some balance of power and accountability. With this foundation, students can then debate what aspects
of the firm would benefit most through external advice and counsel: legal, management consulting, finance,
emerging markets, key customers, people with experience in rapidly growing firms, etc. These areas suggest the
type of individuals and what experience and contacts should be sought out as new directors.
2. Develop arguments as to why and why not Lex Luthor should be appointed as chairperson of the
new board.
The case where a CEO also occupies the chairperson position (referred to as CEO duality) is common, but
increasingly controversial. The best answers will note that there are certain trade-offs between this choice, and
hence no easy answer. On the one hand, a CEO/chairperson may be able to execute strategies quicker given his
or her added power and knowledge accrued from the role as CEO. Such nimble decisiveness may be very
important for firms in fast cycle industries like software. On the other hand, duality dulls the clean distinction
between ownership (through board representation) and managerial control, and increases managerial power and
hence the threat of opportunistic behavior.
3. How should the board design the executive compensation scheme for Luthor, Kool, and Maxter?
Most CEOs in fast-cycle industries also serve as chairperson. This means that the power balance and
incentive alignment may have to be achieved through the level and structure of compensation. Regardless,
speaking only to CEO compensation, the level of cash compensation would need to be set high enough to attract
a talented manager. Students may also agree that the bulk of compensation should come in the form of bonuses,
stock options, or other long-term incentive pay since such a structure will reward the CEO for increases in
shareholder value. This is the essential objective of compensation, the alignment of executive and shareholder
interests. The students should make the point that a compensation plan (such as bonuses and other incentive
pay) that is based on short-term outcomes encourages the top managers to neglect the long-run welfare of the
firm and its shareholders, and even to manipulate the company's financial results.
Quiz 11: Organizational Structure and Controls
1. Discuss the difference between strategic and financial controls.
Strategic Controls: Subjective criteria
- Concerned with examining the fit between:
- what the firm might do (opportunities in its external environment).
- what the firm can do (competitive advantages).
- Evaluate the degree to which the firm focuses on the requirements to implement its strategy
Financial Controls: Objective criteria
- Accounting-based measures
- Return on investment
- Return on assets
- Market-based measures
- Economic Value Added (EVA)
Strategic controls (largely subjective criteria) and financial controls (largely objective criteria) are the two
types of organizational controls used to implement a strategy. Both controls are critical, although their degree of
emphasis varies based on individual matches between strategy and structure.
2. Describe the three major types of organizational structure and their appropriate use.
Organizational structure specifies the firm’s formal reporting relationships, procedures, controls, and
authority and decision making processes. Essentially, organizational structure details the work to be done in a
firm and how that work is to be accomplished. Organizational controls guide the use
of strategy, indicate how to compare actual and expected results, and suggest actions to take to improve
performance when it falls below expectations. A proper match between strategy and structure can lead to a
competitive advantage anticipate the need for structural change and quickly modify structure to better
accommodate the firm’s strategy when evidence calls for that action.
3. Discuss the organizational structures used to implement the different business-level strategies.
4. Define the three major dimensions of organizational structure: specialization, centralization, and
formalization. How do these dimensions vary in organizations implementing the cost-leadership, differentiation,
and the cost-leadership/differentiation strategies?
5. Discuss the organizational structures used to implement corporate-level strategies.
6. Describe the organizational structure associated with a firm that pursues an unrelated diversification
strategy.
7. Describe the organizational structures used to implement the three international strategies.
8. Describe the organizational structures used to implement cooperative strategies, giving attention to the
role of the strategic center firm.
Case Scenario 1: Compliance, Inc. Compliance, Inc., (CI) conducts clinical human and animal trials
for the pharmaceutical and biotechnology industries. Revenues are split evenly between early and late
drug development services. While the bulk of its business is conducted in Europe and the U.S. (10 and 17
subsidiaries, respectively), CI also has subsidiaries in Africa, Latin America, Asia, and Australia.
Historically CI operated under a multidomestic strategy, owing to the fact that the clinical testing
industry was geographically fragmented to meet the diverse needs of the many strong local
pharmaceutical companies and distinct regulatory environments. CI's organizational structure truly
reflected the autonomous character of each country's businesses. Many of the country managers have
been with CI for over a decade, and have a great deal of discretion over the activities of their home-
market businesses. However, globalization of the regulatory environment (both global and local
standards), globalization of the biotechnology firms (increasing the geographic scope of their operations),
and tremendous consolidation in the pharmaceutical industry (reducing the number of pharmaceutical
industry participants to only a handful of major global companies) caused CI to question its
multidomestic strategy. Consequently, the firm has begun its transition to a transnational strategy. -
(Refer to Case Scenario 1)
1. What type of organizational structure was likely to have been in place under CI's multidomestic
strategy?
2. What type of organizational structure will likely be needed for its transnational strategy? What impact
will this have on the location of particular value chain activities?
3. What obstacles is CI likely to encounter as it attempts to change its structure to support the transnational
strategy?
Case Scenario 2: Palmetto. Palmetto was an early pioneer of personal data assistants (PDAs) and
dominates that market space (in terms of market share) with its core product, the Palmetto Pidgy.
Because this product category was entirely new to the market, Palmetto had to internally develop the
hardware and software sides of the business, and today is both a manufacturer of PDAs and a
programmer and licensor of its PDA operating system software. Recently however, the hand-held device
maker's performance has taken a dive as a result of slumping sales and costly inventory problems.
Palmetto has also had difficulty coordinating its software and hardware businesses, in part due to the
near absence of a coherent structure and the differing economics underlying the two. Specifically,
hardware for PDAs is increasingly a cost-based business, while software is a highly differentiated one.
While Palmetto is doing pretty well in both businesses, its own resource base does not allow it to compete
any differently than that proscribed for other industry participants (i.e., competes on cost with hardware
and features with software). In addition to these fundamental differences, new large entrants are entering
both the equipment (e.g., Sony) and software (e.g., Microsoft) sides of its business, putting further
pressure on margins. Management has decided that it is unable to focus on the complexities of each of
these businesses so it is opting to break Palmetto into two separate, independent public companies - Pal,
Inc. will be devoted to hardware and Mettolink, Inc. will be devoted to software. -(Refer to Case Scenario
2)
1. What basic structural form would you recommend for both Pal and Mettolink? What must each firm be
careful to avoid with this structure?
2. How would the implementation of this structure differ for Pal?
3. How would the implementation of this structure differ for Mettolink?
Quiz 12: Strategic Leadership
https://quizlet.com/85357941/strategic-management-ch-12-essay-flash-cards/
1. What is strategic leadership, who has primary responsibility for strategic leadership, and what
are the five key strategic leadership actions?
Strategic leadership is the ability to anticipate, envision, maintain flexibility, and empower others to create
strategic change. The CEO has primary responsibility for strategic leadership, which is shared with the board of
directors, the top management team and divisional general managers. The five key strategic leadership actions
are: determining a strategic direction, effectively managing the firm's resource portfolio, sustaining an effective
organizational culture, emphasizing ethical practices, and establishing balanced organizational controls.
2. What is a top management team, and how does it affect a firm's performance and its abilities to
innovate and design and implement effective strategic changes?
The top management team is composed of the key managers in the organization who are responsible for
selecting and implementing the firm's strategy. Typically, the top management team includes all officers of the
firm (defined by the title of vice president or above) and/or those who serve as a member of the board of
directors. Team characteristics have been shown to affect the strategy of the organization. A heterogeneous top
management team is composed of individuals with varied
functional backgrounds, experiences, and education. A homogeneous team's members are similar to one
another in characteristics and experiences. A heterogeneous team is more likely to formulate an effective
strategy because of its varied expertise and knowledge. Additionally, heterogeneous top management teams
have been shown to positively affect performance. In particular, heterogeneous teams positively affect
innovation and strategic change in firms. But, heterogeneous teams are less cohesive than homogeneous teams
because of communication difficulties, and it is more difficult for heterogeneous teams to implement strategies.
Consequently, a heterogeneous top management team must be managed effectively to use the diversity in a
positive way.
3. Discuss how the managerial succession process and the composition of the top management team
interact to affect strategy.
Internal labor markets represent the opportunities for employees to take managerial positions (including the
position of CEO) within a firm. The external labor market is the collection of career opportunities for managers
in firms outside of the one for which they currently work. CEOs may be selected from internal or external
candidates. Internal CEO selection is preferred by employees and by those who wish the firm to continue in its
present strategies. External CEO succession is considered a sign that the board of directors wants change.
Internal CEOs are less likely to seek change in the firm's strategy than external CEOs. It is important to note
that the source of the CEO (from the internal or external labor market) and the top management team's
composition interact to affect the likelihood of strategic change.
If a firm hires a new internal CEO and has a homogeneous top management team, it
is unlikely that the firm's strategy will change. If the firm employs a new internal CEO but has a
heterogeneous top management team, it will probably continue the current strategy, but innovation will be
encouraged. If the top management team is homogeneous, but an external CEO is chosen, the situation will be
ambiguous. Finally, if the top management team is heterogeneous and an external CEO is chosen, strategic
change is likely.
4. Define human capital and its importance to the firm's success.
Human capital represents the knowledge and skills of the firm's entire workforce.
Effective strategic leaders view human capital as a capital resource that requires investment rather than as a
cost to be minimized. It is thought that people are the organization's only truly sustainable source of competitive
advantage. So, effective human resource management practices are necessary to successfully select and use
people to attain the firm's goals. Not only must future leaders be trained, but the entire workforce must be able
to learn continuously to build skills and knowledge that lead toward innovation. Layoffs can be disastrous
because they strip skills and knowledge from the firm, leaving remaining employees unable to perform their
tasks effectively.
5. What is organizational culture? What must strategic leaders do to develop and sustain an
effective organizational culture?
Organizational culture is the set of ideologies, symbols, and core values that is shared throughout the
organization and that influences the way the firm conducts its business. An organization's culture can be a
source of competitive advantage. It is more difficult to change a firm's culture than to sustain it. But effective
strategic leadership recognizes when a change in a firm's culture is necessary. Incremental changes to the firm's
culture are typically used to implement strategies. Sometimes radical changes are used to support strategies that
differ from the firm's historical pattern. Shaping and reinforcing change in an organization's culture require
communication and problem solving, selection processes that find people with the right values, effective
performance appraisals focused on goals reflecting the new culture, and reward systems that reward behaviors
reflecting the new core values. Change occurs only when it is actively supported by the CEO, other top
managers, and middle management.
6. As a strategic leader, what actions could you take to establish and emphasize ethical practices in
your firm?
Ethical practices should be institutionalized within the organization by being an integral part of the
organizational culture. Sustaining an effective organizational culture is one of the key leadership actions and
one aspect of an effective culture is that it promotes ethical behavior in the organization. Strategic leaders also
develop explicit codes of conduct and provide ethics training to disseminate those codes. Examples of specific
actions taken by strategic leaders to develop an ethical organizational culture include: (1) establishing and
communicating specific goals to describe the firm's ethical standards (e.g., developing and disseminating a code
of conduct), (2) continuously revising and updating the code of conduct, based on inputs from people
throughout the firm and from other stakeholders (e.g., customers and suppliers), (3) disseminating the code of
conduct to all stakeholders to inform them of the firm's ethical standards and practices, (4) developing and
implementing methods and procedures to use in achieving the firm's ethical standards (e.g., use of internal
auditing practices that are consistent with the standards), (5) creating and using explicit reward systems that
recognize acts of courage (e.g., rewarding those who use proper channels and procedures to report observed
wrongdoing), and (6) creating a work environment in which all people are treated with dignity.
7. What are organizational controls? Why are strategic controls and financial controls important
aspects of the strategic management process?
Organizational controls are the formal, information-based procedures used by managers to maintain or alter
patterns in organizational activities. Controls provide the parameters within which strategies are to be
implemented, as well as forming guidelines for corrective actions when adjustments are required. There are two
main types of controls: financial and strategic. Financial controls focus on short-term financial outcomes.
Strategic controls focus on the content of strategic actions. Financial controls give feedback about the outcomes
of past actions. Strategic controls focus on the drivers of the firm's future performance. Emphasizing either
financial or strategic controls has important implications for the strategic management process. For example,
emphasizing financial controls often produces more short-term and risk-averse managerial actions because
financial outcomes may be caused by events beyond the managers' direct control. In contrast, strategic control
encourages lower-level managers to make decisions that incorporate moderate and acceptable levels of risk
because outcomes are shared between the business-level executives making strategic proposals and the
corporate-level executives evaluating them.

Case Scenario 1: The Walt Disney Company The Walt Disney Company was founded as a cartoon
studio in 1923 by Walt Disney and his brother Roy with a $500 loan from an uncle. In the early 1920s,
cartoonist Walt Disney visited New York to pitch his idea for a cartoon rabbit called Waldo. During that
trip, through a complicated series of events, Disney lost the rights to develop Waldo. On the train-ride
back to California he spoke with his wife about the importance of coming home with some alternative
character. "I can't come back to our office and tell them I've lost Waldo," he bemoaned. This hardship
inspired Disney to develop a new character, Mickey Mouse, and release the world's first fully-
synchronized sound cartoon, "Steamboat Willie" (starring, of course, Mickey Mouse). Disney's creative
genius was now coupled with a fierce instinct to protect and control his creative output. Never again
would he lose "Waldo." Consequently, the Walt Disney Company was pushed by Walt to tirelessly create
timeless and universal entertainment, consistently innovate and take risks to deliver that entertainment,
stress a vision of being the provider of choice of quality family entertainment, and maintain rigorous
control over the quality of customers' experiences with Disney products and its image. Such a personal
passion for control led the Walt Disney Company into theme parks because Disney did not want Mickey's
reputation sullied by the dirty, cheap theme parks that littered the land during those days. All films had
to be new and of the highest quality animation (taking a minimum of five years to create, including hand-
painted backgrounds); sequel films were not tolerated. Walt's vision and risk taking propensity led him
in the early 1960s to buy 43,000 acres in Florida (now Walt Disney World), betting the company's future
on a high-risk, uncertain venture. Amidst such a flurry of activity, Walt Disney died just before
Christmas 1966, and the company was literally stopped dead in its tracks. Walt Disney's blueprint was
being followed to the letter, but no further (Walt Disney World opened in 1971). No "new" creations were
undertaken until 1982, when the company finally launched such businesses as the Disney Channel,
Touchstone, and their home video business. Had it not been for the appointment of Michael Eisner as
Disney's new CEO in 1984, the company would likely not have survived its perilous financial situation
and stifled creativity. Eisner returned the company to its roots of family entertainment and values of
quality, fairness, creativity, entrepreneurialism, and teamwork. -(Refer to Case Scenario 1)
1. What value-creating legacy did Walt Disney leave to the Walt Disney Company?
2. To what extent had the Walt Disney Company become a reflection of Walt up to the time that he died in
1966?
3. Why do you think the Walt Disney Company had so much difficulty being innovative in the decades
following Walt's death?

Case Scenario 2: Yepsen Timber Farms, Inc. Yepsen Timber Farms, Inc., (YTF) was started around
1933 by Danish immigrants. The firm's primary operations were timber harvesting on several thousand
acres in Oregon acquired in part under the Homestead Act, and in part through direct purchase. The
firm was founded, initially as a partnership, between brothers Mogens and John (Jack) Yepsen. The
Yepson brothers were among the first four graduates at Oregon Agricultural College (now Oregon State
University), worked for the forest service and private industry in Oregon for a number of years, then quit
their respective jobs to manage the forest they had been developing for a number of years. While timber
is considered a low-tech type business, Mogens and Jack were very innovative from the standpoint that
they established "tree farms," that is, harvesting then replanting acreage so that it would yield timber on
a sustainable basis. At the time, and in certain parts of the world to this day, timber lands were typically
"clear cut" where all the trees were stripped from a property, then the timber harvester simply moved to
another parcel. This practice left thousands of acres barren, and often damaged valuable animal habitats
and watershed. The brothers also introduced hybrid Pine and Douglas Fir trees that grew considerably
faster than the native forest stock. These factors allowed them to grow trees that would be ready for
market in 25 years, about half the time of that required to grow native trees. The brothers' idea about
regeneration, care for the environment, and hybridization defined the YTF business. Never would land be
harvested faster than it could replenish itself, or in a manner that threatened habitats or watersheds.
Eventually, Mogens and Jack passed on and their only surviving children, Marjorie, Mary Jane, Burton,
and Betty inherited the property. Two of these heirs took a strong interest in further building the
portfolio of Oregon properties, and also converted the holdings to an S-Corp. to allow for the distribution
of ownership and earnings to their own children. Under their guidance, YTF was tremendously successful
and garnered much community acclaim for its sustainable farming practices. Now, the four siblings are
in their 70s and few of their children have expressed much interest in managing the extensive portfolio of
timber holdings. Among those that have expressed an interest, some are very knowledgeable about
forestry, while others have a track record of incompetence and self-promotion. At the same time,
ownership is now spread among some 40 children, nieces, nephews, and grandchildren of the four
siblings. Many of these individuals' only interest in YTF is the annual dividend check they receive. -
(Refer to Case Scenario 2)
1. What culture did Mogens and Jack nurture in YTF?
2. How important is this culture to the future success of YTF?
3. What must be done to continue the viability of YTF as a sustainable timber farm?
Case Scenario 3: Zachary, Wesley & Partners. Zachary, Wesley & Partners (ZW&P) is a leveraged
buyout (LBO) firm that specializes in friendly buyouts of mid-sized U.S. retailing and manufacturing
firms. ZW&P shuns turnarounds and hostile takeovers; its typical deals retain the existing management
team and provide extensive funding for what is perceived to be an already sound strategy. It focuses on
this type of firm because the partners have good contacts in retailing and manufacturing and they are
typically able to avoid bidding wars when the LBO is negotiated. The firm has been immensely profitable
over the years, in part due to the very extensive and selective due diligence process used to winnow down
the list of prospective targets. Fewer than one out of one hundred candidates are even approached, and
only a fraction of these passes further screens in the LBO negotiations. The resulting profitability has, in
turn, given ZW&P a strong reputation in the financial community for successful deals, and among
managers for being able to put together needed financing with good business plans. -(Refer to Case
Scenario 3)
1. What are this firm's core resources and capabilities?
2. Where are these core resources likely to be located in the firm?
3. What factors may threaten the ability of ZW&P's resources and capabilities to generate continued
success?
Quiz 13: Strategic Entrepreneurship
1. Define the three types of innovative activity. Which is the most critical activity for U.S. firms?
Firms engage in three types of innovative activity: (1) invention, which is the act of creating a new good or
process, (2) innovation, or the process of creating a commercial product from an invention, and (3) imitation,
which is the adoption of similar innovations by different firms. Invention brings something new into being while
innovation brings something new into use.
2. What is the importance of international entrepreneurship?
International entrepreneurship, or the process of identifying and exploiting entrepreneurial opportunities
outside the firm’s domestic markets, has become important to firms around the globe. Evidence suggests that
firms capable of effectively engaging in international entrepreneurship outperform those competing only in their
domestic markets.
3. Describe the three strategic approaches used to produce and manage innovation: internal
corporate venturing, cooperative strategies, and acquisitions.
Three basic approaches are used to produce innovation:
(1) internal innovation, which involves R&D and forming internal corporate ventures, (2) cooperative
strategies such as strategic alliances, and (3) acquisitions. Autonomous strategic behavior and induced strategic
behavior are the two forms of internal corporate venturing. Autonomous strategic behavior is
a bottom-up process through which a product champion facilitates the commercialization of an innovative
good or service. Induced strategic behavior is a top-down process in which a firm’s current strategy and
structure facilitate the development and implementation of product or process innovations. Thus, induced
strategic behavior is driven by the organization’s current corporate strategy and structure while autonomous
strategic behavior can result in a change to the firm’s current
strategy and structure arrangements.
4. Discuss the differences between autonomous strategic behavior and induced strategic behavior.

Induced Strategic Behavior Autonomous Strategic Behavior

Most large companies employ this. Employees are allowed to think


Planned form of influence upon the creatively about innovation outside of
workforce to come up with innovative their present products. They can think
thinking around their present product. about products that they like to create

Nature of screening Formal screening managers define a market


process diverges from the organization's
strategy

activation of the manager defines the market that is in Informal network that assesses
strategic decision line with the org's strategy new ideas
process

Type of innovation incremental to present products major - whole new product lines

Nature of consistent with organizational work Autonomous departs from work


communication flow in early stages of the decision

Major Actors formal as prescribed by the informal network and furthered


organization by a so-called "champion)
Decision Roles roles and responsibilities are well roles and responsibilities are
defined loosely defined in early stages but
become more defined as the project
progresses

Implications for strategic alternatives are considered commitment to a particular


Strategy and a commitment to a particular strategy emerges during the early
strategy evolves stages as the project progresses
through sponsorship of the "Product
Champion"

5. Discuss the methods an organization can use to facilitate cross-functional integration.


6. Discuss the potential benefits and disadvantages of innovation through cooperative strategies.
7. Discuss the benefits and risks of acquiring another firm to gain access to innovations.
Case Scenario 1: Fear Not. Wim Vijkland was trained as an engineer in the Netherlands and, after
college, worked several years in the Chinese operations of Philips Electronics and then Unilever. Between
employers he returned home for several years to complete an MBA from Tiburg University in the
southern Netherlands. His work gave him hands-on experience with overseas production, and rich sets of
contacts in Mainland China and distribution channels in Europe and the U.S. Wim has noted that many
small and mid-sized European and U.S. manufacturers are interested in and would benefit from the low-
cost Chinese production environment. Contrary to external stereotypes, he also believes that a Chinese
factory can produce products that meet the most demanding technical and quality specifications met by
manufacturers in more developed economies. At the same time, Vijkland understands that "foreigners"
are generally reluctant to manufacture precision products in China for fear that the underlying
proprietary technologies will be bootlegged and sold to competitors or outright copied. In an attempt to
capitalize on this opportunity, Wim quit his job with Unilever and entered into a partnership with Sulin
"Cathy" Liu, a local Beijing entrepreneur with whom Wim has worked extensively in the past. Cathy has
a Ph.D. in physics from CalTech in California and an MBA from Hong Kong University of Science and
Technology. They have dubbed their partnership FearNot, and organized it as a limited liability
corporation (LLC). Their plan is to set up duty-free manufacturing zones in which they develop mini-
factories that operate under their ownership and production guidance, while at the same time creating a
firewall between the clients' proprietary production processes and the open Chinese market. It is the
partners' hope that this combination of intellectual property protection and low-cost overseas production
will provide U.S. and European firms an incentive to enlist FearNot's services. -(Refer to Case Scenario 1)
1. What resources do Wim and Cathy bring to their venture?
2. What resources appear to be missing?
3. What should FearNot focus on in its first months of operation?
Case Scenario 2: Compliance, Inc. Compliance, Inc. (CI) conducts clinical human and animal trials
for the pharmaceutical and biotechnology industries. Revenues are split evenly between early and late
drug development services and the firm is a leader in the laboratory technologies needed for such testing.
One of CI's internal quality managers, Sharon Kline, has approached the CEO with a new business
proposal. She would like to see the firm take one of its in-house software programs and develop it as a
leading-edge commercial product for three specific target markets - medical care providers, payers of
medical care, like insurance companies, and suppliers to medical care providers, like pharmaceutical
companies. The features of the software are easy to use and include electronic distribution, data
harvesting, and robust reporting capabilities. With this software Sharon believes that medical care
providers will be able to collect data to market to and negotiate contracts with payers or employers,
profile performance of individual physicians or practice sites, identify best clinical practices, generate
reports that satisfy regulatory or accreditation requirements for provider sites, and supply professional
societies with data for influencing payer and government policies. Another target market, insurance
companies and other medical services payers, will be able to use the software to profile performance of
individual physicians or practice sites, identify best clinical practices, generate reports that satisfy
regulatory or accreditation requirements, and collect data to market to and negotiate contracts with
employers. Finally, the software will allow suppliers to medical care providers to assess how products
perform compared to competitor products, assess outcomes in real-world compared to clinical trial
settings, obtain information on provider-specific practice patterns, determine whether products are being
used correctly, get "face-time" with physicians and HMOs, obtain information on product switching
behavior, offer providers a value-added service, and meet FDA post-marketing surveillance
requirements. CI has never launched such a product before and, even if successful, software is a very
different product than the clinical trials services it provides now. The CEO must determine how to build
and manage this new business for CI. -(Refer to Case Scenario 2)
1. Does CI's launch of the software product better fit the notion of autonomous or induced strategic
behavior?
2. Where does it appear that autonomous strategic behavior ends and induced strategic behavior begins in
the software situation at CI?
3. What actions would you recommend that the CEO undertake to ensure the success of the software
venture?
Case Scenario 3: Bunnywac. Bunnywac is a global producer and seller of batteries for consumer
electronics, and competes primarily by providing battery products equal in performance at a lower price.
The worldwide battery industry suffers from issues of overcapacity and commoditization, brand
segmentation and proliferation, the growing strength of global retailers, and the low-cost threat of new
entrants from Asia. Bunnywac has grown quickly into one of the leading players in the battery industry
primary through horizontal acquisitions, and is now counted among the top four companies in North and
Latin America. Its presence in Europe and Latin America is negligible. A key factor in Bunnywac's rapid
growth is its technology outsourcing arrangement with Mats. Mats is one of Japan's largest technology
holding companies and Bunnywac's core battery technology is licensed from Mats. Bunnywac's license
with Mats expires soon and it is concerned that Mats will not renew it, or will renew it only for a
substantial price premium. Consequently, Bunnywac's CEO is exploring the possibility of developing its
core technology in-house. -(Refer to Case Scenario 3)
1. Does Bunnywac's development of its core battery technology better fit the notion of autonomous or
induced strategic behavior?
2. What aspects of Bunnywac's development of its core battery technology do not fit the notion of strategic
behavior that you identified above?
3. How will Bunnywac's structure likely need to change if it does begin developing its core battery
technologies in-house?

You might also like